CTET February 2015 Paper-II Previous Year Paper

CTET February 2015 Paper-II

Q. 1 Which of the following statements best summarizes the relationship between development and learning as proposed by vygtosky ?

A. development is independent of learning

B. development process lags behind the learning process

C. development is synonymus with learning

D. learning and development are parallel process

 

Q. 2 What is the major criticism of Kohlbergs theory ?

A. Kohlberg created a theory without any empirical basis

B. Kohlberg proposed that moral reasoning is developmental

C. Kohlberg did not account for cultural differences in moral reasoning of men and women

D. Kohlberg did not give clear cut stages of moral development

 

Q. 3 In a learner-centered classroom , the teacher would .

A. encourage children to compete with each other for marks to facilitate learning

B. demonstrates what she expects her students to do then give them guidelines to do the same

C. employ such methods in which the learners are encouraged to take initiative for their own learning

D. use lecture method to explain key facts and then assess the learners for their attentiveness

 

Q. 4 According to garderns theory of multiple intellligence the factor would contribute most for being self aware would be

A. musical

B. spiritual

C. linguistic

D. intrapersonal

 

Q. 5 A major differences between the perspectives of vygtosky and piaget pertains

A. their critque behavioristic princples

B. the role of providing a nurturing environment of children

C. their views about language and thought

D. their conception of children as active constructors of knowlegde

 

Q. 6 a lot of debate sorrounds whether girls and boys have specific sets of children due to thier genetic materials. which one of the following are you most likely to agree withh in this context ?

A. girls are socialized to be caring while boys are discouraged to show emotions such as crying.

B. after puberty boys and girls cannot play with each other since thrir interests are complete opposites

C. all girls inherit talent for arts while boys are genetically programmed to be better at agreessive sports

D. boys cannot be caring because they are born that way

 

Q. 7 A teacher wants to ensure that her students are motivated intrinsically She would

A. Specify uniform standards for acheivement for all children

B. forcus on the process of learning of individual children rather than on the final outcome

C. offer tangable rewards

D. plan learning activities with encourage convergent thinking

 

Q. 8 Failure of a child to perform well in classtests leads us to beleive that

A. assessment i objective and can be used to clearly identify failures

B. There is need to reflect upon the syllabus ,pedagogy and assessment process.

C. some children are deemed to fail irrespective of how hard system tries

D. children are born with certain capabilities and defiects

 

Q. 9 There are few children in your class who makes errors .which of these is most likely to be your analsys of the situation ?

A. The children have poor intelligence

B. the children are not inetrested in studies and want to create indiscipline

C. the children should not have been promoted to your class

D. the children have not yet gained conceptual clarity andthere is need for you to reflect on your pedagogy

 

Q. 10 A student hightlights the main points in a ,chapter , draws a visual representation and poses questions that arise in her mind at the end of the chapter . She is

A. trying to regulate her own thinking by organisation of ideas

B. trying to use strategies to maintanance rehersal

C. ensuring observational learning

D. trying to use method of loci

 

Q. 11 How can a teacher help students become better problem solvers ?

A. By giving children a variety of problems to solve and support while solving them

B. By encouraging children to look for answers to the problems in the textbook

C. By providing correct solutions to all the problems they pose to students

D. By giving tangible rewards for solving problems

 

Q. 12 Of the following statemnets which one do you agree with ?

A. learning is completey governed by external stimuli

B. learning cannot take place unless it is assessed externally in terms of marks

C. learning has taken place only if it is evident in behaviour

D. learning takes place in soico culteral contest

 

Q. 13 which one of these is principle of child development ?

A. development occurs due to interaction between maturition and experience .

B. experience is sole determinant of development

C. development is determined by reinforcement and punishment

D. development can accurately predict the pace of intelleignet child

 

Q. 14 In the context of nature -nature debate which one of the following statements seem appropriate to you ?

A. A child is like a balnk state whose charecter can be moulded by the environment in any shape

B. Environmental influences only have a little value in shaping up childs behaviour which is primarly genetically determined

C. heredity and environment are inseperably interwoven and both influence development

D. Children are genetically predisposed to what they would like to irrespective of whatever environment they are grown in

 

Q. 15 Socialization is a process of :

A. acquiring values, beliefs and expectations

B. assimilation and accomidation

C. learning to critique the culture of society

D. socializing with friends

 

Q. 16 Piaget proposes that pre-operation children are unable to conserve. He attributes this inability to which one of the following factors?

A. inability of hypothetico-deductive reasoning.

B. Personal fable

C. Irreversibility of thought

D. lack of high-level abstract reasoning.

 

Q. 17 According toPiagets theory children learn by

A. memorizing information by paying due attention.

B. scaffolding provided by more able members of the society

C. processes of adoptation

D. changing their behaviour when offered appropriate rewards.

 

Q. 18 According to Vygotsky, zone of proximal development is

A. zone demarking the support offered by the teacher

B. the gap between what the child can do independently and with assistance

C. the amount and the nature of the support provided to the child to achieve her potential.

D. what the child can do on her own which cannot be assessed.

 

Q. 19 A teacher in a multi-cultural classroom would ensure that the assessment consider the following;

A. Reliability and validity of her assessment tool

B. Expectation of the school administration by complying with the minimum levels of learning

C. Standardization of the assessment tool

D. Socio-cultural context of her students

 

Q. 20 An upper primary school constructivist classroom would foresee the following role of students in their own assessement

A. Make detailed guidelines for how marks would be correlated to students achievement and prestige in class.

B. Students would be the sole determenants of their own assessments.

C. Students would plan for assessment with the teacher.

D. Denying the assessment has a role in teaching and learning.

 

Q. 21 The rationale behind incluisve edocation that is

A. society heterogenous and schools need to be inclusive to cater to heterogenous society

B. we need to take pity on special children and provide them access to facilities .

C. it is not cost -effective to provide for seperate schools for special children .

D. the benchmarks for performance of each child should be uniform and standalized

 

Q. 22 Which one of the following would be most effective way to identify a creative child by the student ?

A. observing how child interacts with peers in team task.

B. administering standarlized intelligence tests

C. giving objective type tests

D. detailed observation of the child especially when she solves the problems

 

Q. 23 A teacher can effectively respond to the children from disadvantged sections of society by 

A. telling the other children to co-operate with the disadvantaged children and help them learn the ways of school .

B. reflecting on the school system and herself about various ways in which biases and sterotype surfaces .

C. ensuring that the children do not get a chance to interact with each other of their chances of being bulllied .

D. sensitizing the disadvantaged children to the norms and structures of schools so that they can comply with those

 

Q. 24 Research has pointed out that several levels of discrimintaion exist in schools .Which of these is not an example of discrimination at upper primary level ?

A. Many teachers use only lecture methods to teach

B. Dalit children are made to sit seperatly during mid daymeals

C. Girls are not encouraged to take maths and science

D. Teachers have low expectations of children from lower econmic strata

 

Q. 25 Which of these is a charecterstic of a child with learning disability ?

A. An IQ below 50

B. Bulleying other children and engaging in aggressive acts

C. doing the same motor action repeatedly

D. Difficulty in reading fluently and reversing ideas

 

Q. 26 Which of the following statements best describes why children should be encouraged to ask questions in the class ?

A. Questions increase the curiosity of the children

B. Questions take learning forward by interactions and lead to conceptual clarity .

C. Children need to practise their language needs

D. Children can be made to realise that they lack intelligence by making them think of all the things they dont know about

 

Q. 27 Which of the following assessment practises will bring out the best in students ?

A. When students are required to reproduce facts as tested via multiple choice questions

B. When conceptual change and students alternative solutions are assessed thorugh several different methods of assessment

C. When the marks obtained and the position secured by the student in the class are the ultimate determinants of succcess

D. When the emphasis laid upon positive correlation between test scores and student ability

 

Q. 28 The amount and type of scaffolding of a child would change depending upon on the :

A. mood of the teacher

B. reward offered for the task

C. levels of child performance

D. childs innate abilities

 

Q. 29 As an upper primary school mathematics teacher you believe that :

A. students errors provide insights about their thinking

B. not all the children have the ability to study mathematics in upper primary school

C. boys will learn mathematics without much effort since they are born with it and you need to pay more attention to the girl

D. students need to possess procedural knowledge even if they dont understand conceptual basis

 

Q. 30 Which of these statements do you agree with ?

A. A childs feels beacuse the government is not giving enough technological sources in schools

B. A child’s failure can be attributed directly to the genetic material he/she has acquired from his/her parents

C. A child’s failure is a reflection on the system and its inability to respond to the child

D. A child’s failure is primarly due to the lack of parents education and economic status 

 

Q. 31 Which one of the following is not a component of maps ?

A. size

B. symbols

C. distance

D. direction

 

Q. 32 What is the angle of inclination of the earths axis with its orbits place ?

A. 23(1/2)°

B. 45°

C. 66(1/2)°

D. 90°

 

Q. 33 The janpad panchayat has

A. many zila parishads under it

B. many municipal wards under it

C. many gram sabhas under it

D. many gram panchayats under it

 

Q. 34 Consider the following statements A and B on social advertisements and choose the correct answer

A. Social advertisements are advetisements that have larger messages for the society

B. Social advertisements are made only by the state

A. Both A and B are false

B. Both A and B are true

C. A is true B is false

D. A is false B is true

 

Q. 35 Which one of the following states in India has the famous kalamkari print ?

A. Tamil Nadu

B. Karanataka

C. Andhra Pradesh

D. Punjab

 

Q. 36 Which one of the following tribes is only confined to Uttarakhand ?

A. Gonds

B. Mala Irular

C. Bhil

D. Bhotia

 

Q. 37 Who among the following rulers were best known for controlling the silk route around 2000 years ago ?

A. kushanas

B. Pandyas

C. Cheras

D. Cholas

 

Q. 38 Under subsidary alliance the East India Company forced the Indian rulers of the states to 

A. Have their independent armed forces ,not get protection by the company , but still pay towards its maintanance .

B. not have their independent armed forces ,get protection by the company without any payment towards its maintanance

C. Have their independent armed forces as well as get protection by the company and pay towards its maintanance .

D. not have their independent armed forces , but get protection by the company ,and pay towards its maintanance .

 

Q. 39 Which one of the following is not a fact about Ahom ?

A. The Ahom state was divided into Clans called Paiks .

B. The Ahom state was very sophisticated .

C. The state of Ahom was suppressing the older political system of the Bhuiyans .

D. The Ahom state dependent on forced labour

 

Q. 40 The term ,Hagiography refers to

A. writing of saint’s biography

B. writing of saint’s autobiography

C. writing of ruler’s biography

D. writing of ruler’s autobiography

 

Q. 41 Which one of the following is a type of tropical grasslands ?

A. Savanna

B. steppe

C. down

D. pampas

 

Q. 42 Consider the following statements A and B ,about food security and choose the correct answer .

A. The ultimate aim of agricultural development is to increase food security .

B. Food security refers to the existence of availabilty of food to all the people in the times of natural calamities ,when there occurs a shortage of food .

A. Both A and B are true

B. Both A and B are false

C. A is true and B is false

D. A is false and B is true

 

Q. 43 Which of the following is not considered as fossil fuel ?

A. Petroleum

B. natural gas

C. fire wood

D. cool

 

Q. 44 Which one of the following may cause tremendeous chnages only on the coastal areas ?

A. Tsunami

B. volcano

C. Tides

D. Earthquake

 

Q. 45 The inner most layer of the earth is mostly made up of :

A. Silica and Iron

B. nickel and alumina

C. Silica andalumina

D. nickel and Iron

 

Q. 46 Which one of the following is the result of depositional work of river ?

A. Manders

B. ox-bow lake

C. waterfall

D. food plain

 

Q. 47 During an earthqauke at what measurement on the ritcher scale do things begin to fall ?

A. 6.0 or higher

B. over 7.0

C. 2.0

D. over 5.0

 

Q. 48 Which one of the following was introduced with the coming of british art in India during colonial period ?

A. mural art

B. oil painting

C. perspective

D. miniatures

 

Q. 49 Around 2000 years ago varanasi was a famous craft centre where shrenis were.

A. Buddhist monesteries

B. association of craft persons and merchants

C. iron ploughshares used for agriculteral production

D. coins that have designs punched on to the metal

 

Q. 50 consider the following statements A and B on manuscripts and inscriptions and choose the correct answer .

A. Manuscripts were usually written on palm leaf or on the specially prepared bark of a tree known as brich

B.Inscriptions were written on relatively hard surfaces like stone and metal.

A. Both A and B are true

B. Both A and B are false

C. A is true and B is false

D. A is false and B is true

 

Q. 51 Which of the following was language of administration under the Delhi Sultans?

A. Persian

B. Hindi

C. Arabic

D. Urdu

 

Q. 52 In Buddism ,Bhoddisattavas were

A. Persons who attended enlightenment

B. Chinese buddhist pilgrims

C. Buddhist scholars

D. Statues of buddha

 

Q. 53 Which one of the following was not a condition to become a member of sabha in a chola empire ?

A. they should be between 25 to 70 years of age

B. they should have their own homes

C. they should have knowledge of vedas

D. they should be owners of land which land revenue is collected

 

Q. 54 The Rowlatt act ,1919 was opposed because .

A. it gave right to seperate electorates to muslims

B. it disallowed indians to from possessing arms

C. it curbed the fundamental rights like freedom of expression and strengthened police powers

D. introduced dyranchy in provinces

 

Q. 55 Many of chanakya’s ideas were written in the book called the

A. Atharvaveda

B. Yajurveda

C. Dharmashastras

D. Arthashastras

 

Q. 56 Garbhagriha , the most important part of some of the earliest hindu temples was , 

A. the place in temples where people could assemble

B. caves elaboratley developed with sculpteres and painted walls

C. the place where the image of the cheif deity was installed

D. the place where religious rituals were not performed

 

Q. 57 Which one of the following grasslands is a hometown of native americans ?

A. Steppe

B. Pampas

C. Velds

D. Prairies

 

Q. 58 Breeding of fish in specially constructed tanks and ponds is known as :

A. Pisiculture

B. Horticulture

C. sericulture

D. viticulture

 

Q. 59 Being Strait is a narrow stretch of shallow water that connects.

A. Pacific ocean and Atlantic Ocean

B. Pacific ocean and Arctic Ocean

C. Pacific ocean and Indian Ocean

D. Atlantic Ocean and Indian Ocean

 

Q. 60 On 21st march and 23rd september ,the whole earth experiences equal days and nights, this is because ,

A. Direct rays of the sunfall on the eqautor and at this position ,none of the poles is titles towards the sun

B. Direct rays of the sunfall on the tropic of cancer and at this position ,one of the poles is titles towards the sun

C. Direct rays of the sunfall on the eqautor and at this position ,one of the poles is titles towards the sun

D. Direct rays of the sunfall on the Tropic of capricorn and at this position ,none of the poles is titles towards the sun

 

Q. 61 What kind of language is used in social science language ?

A. Representative of the dominant groups

B. Discriminatory

C. neutral and sensitive

D. difficult

 

Q. 62 Projects and democratic institutions could be done to show that

A. they can be analysed to identify features and challenges

B. they are as described in the textbooks

C. they are ineffective

D. students must accept their institutions as they are

 

Q. 63 Summative assessment is inapprpriate for the following

A. deciding upon the grade

B. end of the term assessment

C. monitoring the progress of teaching -learning process

D. summarising student learning

 

Q. 64 what are pie diagrams useful for ?

A. Showing features of historic period

B. comparing different amounts

C. showing in a particular amount in relation of the whole

D. showing in pattern

 

Q. 65 Each chapter of the history for class Vi has been introduced by a young boy or a girl .Why do you think this strategy is adopted by the textbook makers ?

A. To show boys and girls can study history

B. To evaluate the content of the chapter

C. To inculcate a spirit of enquiry and iscovery in studnets

D. To show how intriduction can be done

 

Q. 66 You are a peasnat in Alauddin Khalijis or mohammad bin tuglaq reign and you cannot pay taxes demanded by the sultan .What will you do? In the above question what is being encouraged

A. Historical imagination

B. Importance of sources

C. Respect of culture

D. Imporrtance of literature

 

Q. 67 At the upper primary stage , Geography and economics together can help in the following 

A. Developing a perspective of issues of environment ,resources and development .

B. understanding plurailty

C. developing knowledge of places and environment

D. understanding optimal allocation of resources

 

Q. 68 When is formative evaluation conducted?

A. At the end of the session

B. before starting the new unit

C. during teaching -learning process

D. at the end of the unit

 

Q. 69 Biases , discrimination and prejudices in the classroom could be avoided by

A. organising a lecture on humanity

B. ignoring them as children will grow out ofthem one day

C. discussing different dimensions of social relief

D. handling out the preamble of the constitution

 

Q. 70 Teacher B is currently teaching a lesson on history of a region .What do you think B should do ?

A. Highlight the similarities and changes in different time periods.

B. Highlight identities of different rulers

C. Encourage students to remember main parts of the periods

D. Highlight the similarities of the present with the past

 

Q. 71 What is the purpose of evaluation ?

A. To identify students

B. to rank children

C. to encourage competition

D. to improve teaching-learning process

 

Q. 72 Historcal films are useful in social science teaching as they .

A. concentrate on individual problem

B. potray history as fiction

C. bring alive various dimensions of a special social setting

D. have good entertainment value

 

Q. 73 What kind of outlook should be there towards social science textbooks?

A. As a document to be memorized

B. As an avenue of further enquiry

C. As a repository of knowledge

D. As a final statement

 

Q. 74 Tawa Matsya sangh , a federation of fishworkers co-opertaives is an organisation fighting for the rights of displaced forest dwellers in the state of

A. uttarakhand

B. Chattisgarh

C. Jharkhnd

D. Madhyapradesh

 

Q. 75 Which of the following fundamental rights is not guarenteed to every arrested person under the criminal law as stated in article 22 of the constitution ?

A. confessions made in police custody can be used as evidence against the custody

B. the right not to be ill treated or tortured during the arrest in the custody

C. the right to be informed at the time of arrest of the offence for which the person is arrested

D. the right to be prepared before a magistrate within 24 hours of arrest

 

Q. 76 The members of legisilative assembly ?

A. elected by the people

B. appointed by the governer

C. appointed by the prime minister

D. appointed by the cheif minister

 

Q. 77 A coalition government implies

A. power sharing between 2 or more poltical parties after elections when no party has been able to get a clear majority .

B. power sharing between state governmet or panchayti raj institutions

C. power sharing between government at centre and state

D. power sharing between executive and judiciary

 

Q. 78 At the upper stage ,social science comprises

A. Geography, history , political science and economics

B. Geography, history , political science and sociology

C. Geography, history , political science and environmental science

D. Geography, history , economics and environmental science

 

Q. 79 The position paper on teaching of social science emphasis on

A. respecting different opinions and examining ideas and practises

B. accept social heirarchies

C. increase the use of textbooks

D. encourgaing memorization

 

Q. 80 Maps and diagrams in a social science textbook are relevant as they

A. add a new dimension and concept

B. are made by experts

C. make a textbook attractive

D. clarify a concept

 

Q. 81 A case study of social movement given to a class for discussion need not incorporate the following :

A. solutions to the movement

B. background of the movement

C. objectives of the movement

D. problem areas of the movement

 

Q. 82 While teaching human resources to class VIII students , what core message would you like to promote ?

A. Importance of people as a resource

B. utilization of resources by human beings

C. distribution of population

D. importance of physical resources

 

Q. 83 Which one of the following is not a function of gram panchayat ?

A. executing government schemes related to generation of employment in the villages 

B. the construction and maintance of the infrastructure of the villages

C. levying and collecting local taxes

D. gram panchayat elects its panchayat secretery

 

Q. 84 Which one of the following does not fall within the purview of civil law ?

A. disputes related to safe of land

B. harassing a woman for dowry

C. filing a divorce case

D. cliaming custody of children

 

Q. 85 Which one of the following statements is incorrect about the judiciary ?

A. Judiciary has a power to strike down laws passed by the parliament

B. the judicial system provides a mechanism for resolving disputes between citizens and the government , but not between state governments

C. the legislature and execuitve cannot interfere in the work of judiciary

D. it plays a crucial role in fundamnetal rights of citizens

 

Q. 86 The right of constitutional remedies

A. allows only linguistic minorities to approach the court if they believe that the cultural and educational rights provided in have been violated by the state

B. allows all the citizens to approach the court if they believe that any of the directive principles of state policy have not been fulfilled by the state

C. allow citizens to approach the court if they believe that any of their fundamental rights hvae been viollated.

D. allows only religious minority to approach the court if they believe that their cultural and educational rights provided in the constitution have been violated by the state

 

Q. 87 The civil rights movements is started in USA

A. to demand eqaul rights and address racial discrimination for all the african -american people

B. to demand voting rights for all the african -american men

C. to demand social security of the african -american senior citizens

D. to demand voting rights for all the african -american women

 

Q. 88 Which of the following statements in not true about the parliament of india ?

A. In the matters dealing with finances , the governement does not require parliaments approval

B. The parliament while in session , begins with a question hour

C. The parliament consists of the President , Lok sabha ,Rajya Sabha

D. The parliament in our system has immense powers because it is the represntative of the people

 

Q. 89 While teaching social science , a teacher must attempt to ,

A. focus upon remembering the details

B. focus upon values , patterns and facts

C. focus upon facts but not on values

D. focus upon values but not on facts

 

Q. 90 Suppose a prescribed social science textbook is difficult for students .What will you do ?

A. provide a variety of supplementary materials

B. dictate summary of each lesson

C. use question answer technique

D. emphasize rote learning

 

Questions: 91 – 99

The real indictment against colonialsm was to be found in the villages of India. There was a rot at the top ,too, in the thousands of young intellectuals trained in english schools for jobs that did not exist in limited civil services .The towns and cities were frothing with unhappy young men cultered and well educated who could find no jobs and were not allowed by the old super structure of empire to create them . But the real proof of evil ,I say again was in misearbale villages .I thought i had seen poverty in china ,Yet when i see indian villages i knew the chinesepeasant was rich in comparission.Only the russian peasant i had seen years before could compare with the indian villagers ,although the russian were very different creature and inferior in many ways. And the children , the little children of the indian villages they tore at my heart: thin, big bellied and all with huge darks eyes !I wondered that any englishmen could look at them and not execute himself .Three hundred years of english occupation and rule and could there be children like this .Yes millions of them !! And the final indicement surely was that the life span in india was only 27 years .No matter then that man married very young so that there could be children as many as possible before he died..I love england remembering all the happy days there ,but in india i saw england i did not know 

 

Q. 91 During the colonial rule ,the villagers suffered because there was/were

A. no schools in the village

B. not enough land for agriculture

C. not enough food for children

D. no jobs in the village

 

Q. 92 Even the city people were not happy because _______ .

A. there were not enough jobs for all

B. the trade was under the colonial control

C. there were a large number of young intellectuals

D. business was not flourishing

 

Q. 93 However,the cities were better off than the villages because ______

A. the young men were happy

B. the children at least wee not hungry

C. all children were highly educated

D. the educated youth got jobs

 

Q. 94 Which one of the following is true ?

A. The russian peasant could compare with the indian peasant .

B. The Indian peasants were superior to the chinese peasants

C. The chinese peasants were better off the russian peasants

D. The Indian peasants were inferior to the russian peasants

 

Q. 95 The writer believes that a/an _______ should feel sorry at the sight of these poor children . 

A. russian

B. Indian

C. chinese

D. englishmen

 

Q. 96 The english rule made India _____

A. technically rich

B. industrially strong

C. culterally strong

D. economically poor

 

Q. 97 The average life of an indian during the british rule was 27 years .The result was that _______.

A. Indians married young

B. Indians did not have many children

C. Indian was full of young people

D. Indians lived a healthy life

 

Q. 98 The towns and cities were frothing with unhappy young men .The underlined phrase means ______.

A. devoid of

B. full of

C. lacking in

D. angry with

 

Q. 99 The final indicment was inevitable .What does the underlined word mean ?

A. award

B. negation

C. accusement

D. punishment

 

Questions: 100 – 105

I want to be with people with submerge in the task ,who go into the fields of harvest and work in a row and pass the bags along , who are not parlour generals and field deserters but move in common rhythm when the food must come in or the fire be put out The work of the world is common as mud Botched ,it smears the hands , crumbles to dust . But the thing worth doing well done has a shape that satisfies ,clean and evident Greek amphores for wine and oil , hopi vases that held corn are put in in museums but you know they were made to be used The pitcher cries for water to carry and the person for work that is real .

 

Q. 100 The poet seems to admire __________.

A. parlour generals

B. field deserters

C. wage earners

D. farm workers

 

Q. 101 What is common between parlour generals and feild deserters ?

A. both love to work

B. both enjoy respect in society

C. neither of them fights

D. both of them love fighting

 

Q. 102 What happens when work with mud gets botched ?

A. It leads to satisfaction

B. hands get dirty

C. no one pays for it

D. it is abondoned

 

Q. 103 The figure of speech used in lines 12-14 is

A. personification

B. alteration

C. irony

D. metaphor

 

Q. 104 Mud in the hands of good craftsmen becomes _____

A. a useful article

B. an expensive article

C. a work of art

D. a museum piece

 

Q. 105 Amphoras, vases and pitchers are metaphors for ______

A. useful human labour

B. pride in wealth

C. items of luxury

D. antique art

 

Q. 106 Anshu is teaching English to class IV students and her class seems to be noisy. She is probably

A. teaching a crowded class

B. not bothered about the noise

C. having group work

D. not be able to manage the class

 

Q. 107 A method is a body of ______ a teacher adopts the teaching learning process

A. techniques

B. pointers

C. printers

D. norms

 

Q. 108 One of the objectives of teaching vocabulary is not to

A. be able to use words in different context

B. enable learners to use the dictionry

C. understand the meaning of words

D. develop active and passive vocabulry

 

Q. 109 one of the obectives of teaching vocabulary is not to

A. be able to use words in different contexts

B. enables learners to use the dictionary

C. understand the meaning of words

D. develop active and passive vocabulary

 

Q. 110 While selecting a reading text for you students , which of the following is learning important.

A. is approprite for their age level

B. it harldy gives space to think ahead hypothesize and predict .

C. the language of the text is comprehnesible to the learners

D. the context of text is acessable to the learners to applu their own background knowledge

 

Q. 111 giving effective feedback to the learners means

A. doagnosing the problems and guiding the learners on how to improve

B. giving approriate time and space for corrections

C. focus on positive aspects as well as negative

D. correcting their mistakes in the class

 

Q. 112 Which one of the following recommended the 3 language formula ?

A. National policy on education 1968

B. National policy on education 1986

C. kothari commission 1966

D. chattopadyaya commission 1985

 

Q. 113 Which one of the following a teacher should not use while initiating the process writing approach ?

A. Dreafting and revising

B. proof reading before final draft

C. brainstorimng

D. dictating notes

 

Q. 114 Point out the figure of speech used in the sentence below

The moon smiled at the stars around her .

A. metaphor

B. oxymoron

C. personification

D. smile

 

Q. 115 Active vocalbulary consists of words which

A. we use frequently in our daily life

B. are difficult

C. we recognise and understand

D. we use occasionally

 

Q. 116 A purposeful collection of students work that demonstates their efforts , progress and acheivement in a given area is called a/ an

A. antecedal record

B. checklsit

C. rating scale

D. portfolio

 

Q. 117 Every class is a mixed ability class so while dividing the students into different groups for a language activity a teacher should

A. give students different tasks according to their strength

B. arrange groups differently for different kinds of activities

C. put weeker and stornger students in different groups

D. mix weeker and stornger groups

Q. 118 constructivist approach learning does not emphasize .

A. that authentic task in a meaningful context should be used

B. that learners construct knowledge for themselves

C. knowledge construction instead of knowledge reproduction

D. teaching as the transmission of knowledge form the enlightened to the unenlightened

 

Q. 119 A learner -centred class should not be one where

A. the learner should do the given excerisize silently as is better for learning to occur

B. the learners are intrinstically interested and are inclined to explore

C. the teacher acts as more as another particpant in the learning process

D. the learner’s personal views and feelings are taken into consideration

 

Q. 120 Summative assessment is

A. assessment in learning

B. assessment as learning

C. assessment for learning

D. assessment of learning

 

Questions: 121 – 129

The nation is proud of its scientists and scholars ,though ,of course many of them could reply that they doubt whether the nation cares for them at all. When asked why many of our best and brightest have gone abroad to make a living , they opine that this is because of nation we have not cared for the talented and meritorious. There is some truth in what they say .However by and large comparison to the situation before independence government assessment has provided a tremendous opportunity for higher education.If today Indian scientists ,technologies and scholars in different fields are respected worldwide .,it is because of the education system that we built up. Our excellence is evidence within the confines of the limited opportunities which were available for research and development in the universities and national R&D laboratories .We believe and appeal that scientists and researchers and scholars should shed their pessimism.There are many reasons for it. We know the problems face they ,especially the younger ones ,and also those who are not in positions of power in these institutions the so called middle levels and lower levels .They also have the capability to absorb the knowledge base which is growing at an explosive rate.

 

Q. 121 What is ironic about the pride in our scientists ?

A. They go abroad to make a living

B. They are held in high esteem

C. The nation cares for them

D. they are talented and meriterious

 

Q. 122 What happens to our best scientists ?

A. They get government grants

B. They starting teaching in colleges.

C. They start doing research

D. They don’t get respectable jobs here

 

Q. 123 After Independence how has the situation changed in India .

A. The government is sending scientists abroad

B. Foreign scholars are teaching in our universities .

C. The scientists are given padma awards

D. Our system of higher education has improved a lot

 

Q. 124 Our scientists have proved to be excellent even if ________

A. We have excellent research centres

B. we pay them well

C. we offer them limited oppurtunities

D. we offer them excellent oppurtunities

 

Q. 125 The writer wants our scientists to ______

A. to go abroad to make a living

B. be optimistic in their optimistic

C. become a part of scientific community

D. be pessimitic in their approach

 

Q. 126 Which one of the following is true ?

A. Our scientists are respected all over the world

B. We have the best research facilities in India

C. we care for our scientists

D. our scientist are not talented

 

Q. 127 The writer makes an appeal to ______.

A. neither the middle nor the lower level scoentists

B. both the middle and the lower level scoentists

C. the middle level scientists only

D. the lower level scientists only

 

Q. 128 The phrase at an explosive rate means .______

A. at abnormal speed

B. at normal speed

C. at a great speed

D. With the help of an explosion

 

Q. 129 The word opposite in meaning to unleashed is .______

A. controlled

B. unfastened

C. inexpensive

D. uncontrolled

 

Questions: 130 – 135

My heart grieved when I read the other day a news item that more than 2/3rd of the elderly are being ignore din their twilight years.They are isolated by their own children and are expecting loneliness in their present lives.Most of them say that they are not being respected or given good treatment by their family or society and they are being discriminated against in their old age.Majority of elderly attributed their loneliness to no or little interaction with the family members .With fast changing sociology -economic scenarios ,growing nuclear families and changing value systems ,the need of the elderly have also changed ..The most common problems that the elderly face are -marginalization ,lack of love and affection ,little or no access to medicines ,finding it difficult to secure their life and property . To improve the situation of the elderly ,the government of India may consider enacting a law like many other countries have to make it compulsory for children to maintain and look after their parents and also establish the commission for elderly persons .At the same time the facts remain that the real compulsion has to come out of the conds of affection between parents and children .We would not be wrong in saying that in India the joint family system has made a powerful contribution in cementing the bonds of affection between the old and the young.

 

Q. 130 The twilight years are when one is ______

A. middle -aged

B. old

C. a child

D. young man

 

Q. 131 The elderly do not feel ______

A. respected

B. discriminated against

C. lonely

D. ignored

 

Q. 132 What the elderly do not lack ______.

A. is security of life and proeperty

B. is loneliness in their lives

C. are love and affection

D. are medicines and health care

 

Q. 133 Only _____ can keep the old united in the family .

A. courts

B. social compusions

C. joint family systems

D. governments

 

Q. 134 Which of the following is false ?

A. the old love to live in extended families

B. the old are not very mobile .

C. ones need gets changed when one gets old

D. most old people are financially secure

 

Q. 135 Which word in the passage means same as saddened ?

A. isolated

B. restricted

C. grieved

D. ignored

 

Q. 136 For process writing ,which one of the following points should be followed ?

A. use of personal prononts

B. word limit

C. logical sentence

D. organisation of points

 

Q. 137 Problems of spelling errors of the studnets can be easily solved though ____.

A. dictionary

B. dictation

C. teaching aids

D. textbooks

 

Q. 138 _______ is the pre-requisite of the learning language

A. motivation

B. knowing words and their meaning

C. knowledge of grammar

D. reading words

 

Q. 139 Cognitive code approaches focuses more ___________

A. vocabulary

B. spelling

C. grammar values

D. pronunciation

 

Q. 140 According to which one of the following linguists ,labguage cannot be sub standard may be non standard ?

A. william labov

B. stephen krashen

C. chomsky

D. basil bernstain

 

Q. 141 Which one of the following is not a langauge component ?

A. structure

B. script

C. vocabulary

D. sound

 

Q. 142 Decoding stands for _______.

A. deciphering in sounds in words and sentences

B. only recognising the differences between the sounds

C. failure to understand the giventext

D. passing on message toothers

 

Q. 143 In order to improve the learners pronunciation , the teacher first needs to ____

A. Use record model of sounds

B. show the documentary/films of the language

C. ask them to read more

D. do pronunciation drills

 

Q. 144 Intensive reading stands for ______.

A. reading of enrichment of vocabulary

B. reading of developing literary sensitivy

C. reading the texts in parts of details

D. reading for pleasure

 

Q. 145 Scanning is a reading activity which helps the learners to _____ from /of the text .

A. draw out information

B. develop competency in the linguistic items

C. know the meaning of the words

D. enteric the vocabulary

 

Q. 146 A good listener can ______ .

A. correctly respond to every question asked

B. pronounce each and every word correctly

C. understand how to spell a word

D. recognize how inootation used

 

Q. 147 Constructivist approach to language learning promotes ________

A. a child centred approach

B. a teacher centred approach

C. culture of silence

D. classroom noise

 

Q. 148 ______ is the father of modern linguistics .

A. Bloomsfield

B. Chomsky

C. Chaucer

D. Ferdinand de saussure

 

Q. 149 A Phoneme is a _____ .

A. technique to improve pronunciation

B. sound pattern

C. lexal item

D. single sound unit

 

Q. 150 In order to add language proficiency which one is not essentially required ?

A. efficiency

B. Appropriacy

C. Accuracy

D. Fluency

 

Answer Sheet 
Question123456 78910
AnswerBCCDCABBDA
Question11121314151617181920
AnswerADACACCBDC
Question21222324252627282930
AnswerADBADBBCAC
Question31323334353637383940
AnswerACDCCDADAA
Question41424344454647484950
AnswerACCADDDCBA
Question51525354555657585960
AnswerAAACDCDABA
Question61626364656667686970
AnswerCACCCAACCA
Question71727374757677787980
AnswerDCBDAAAAAD
Question81828384858687888990
AnswerAADBBCAABA
Question919293949596979899100
AnswerCABCDDABCD
Question 101102103104105106107108109110
AnswerCBCAACABCB
Question 111112113114115116117118119120
AnswerACDCADBDAD
Question121122123124125126127128129130
AnswerADDCBABCAB
Question131132133134135136137138139140
AnswerABCDCCBACA
Question141142143144145146147148149150
AnswerBAACAAADDA

CTET February 2015 Paper-I Previous Year Paper

CTET February 2015 Paper-I

Q. 1 Teachers, in order to help learners construct knowledge, need to focus on

A. scores/marks obtained by the learner.

B. involving the learner for active participation.

C. mastering learning of concepts by the learner.

D. making sure the learner memories everything.

 

Q. 2 Giftedness from teacher’s point of view is a combination of

A. High Motivation – High Commitment – High Talent

B. High Ability – High Talent – High Commitment

C. High Talent – High Creativity – High Memory

D. High Ability – High Creativity – High Commitment

 

Q. 3 According to NCF 2005, errors are important because they

A. provide a way to the teachers to scold the children

B. provide an insight into the child’s thinking and help to identify solutions

C. provide space for removing some children from the class

D. are an important tool in classifying students into groups of ‘passed’ and ‘failed’

 

Q. 4 A child’s notebook shows errors in writing like reverse images, mirror imaging, etc. Such a child is showing signs of

A. Learning disability

B. Learning difficulty

C. Learning problem

D. Learning disadvantage

 

Q. 5 Which one of the following is best suited for emotional development of children?

A. No involvement of the teachers as it is the task of the parents

B. Controlled classroom environment

C. Authoritarian classroom environment

D. Democratic classroom environment

 

Q. 6 Teachers need to create a good classroom environment facilitate children’s learning. To create such a learning environment, which one of the given statements is not true?

A. Compliance with teachers

B. Acceptance of the child

C. Positive tone of the teacher

D. Approval of the child’s efforts

 

Q. 7 Given below are some statements about boys and girls. According to you, which one of these is true?

A. Boys should help in household chores

B. All boys should be taught Science and girls, Home Science

C. Girls should help in household chores

D. Boys should help in activities outside the home

 

Q. 8 To be an effective teacher it is important to

A. focus on individual learning rather than group activity

B. avoid disruption caused due to questioning by students

C. be in touch with each and every child

D. emphasize dictating answers from the book

 

Q. 9 Which one of the following is not a suitable formative assessment task?

A. Project

B. Observation

C. Ranking the students

D. Open-ended questions

 

Q. 10 Deficiency in the ability to write, associated with impaired handwriting, is a symptom of 

A. Dyspraxia

B. Dyscalculia

C. Dyslexia

D. Dysgraphia

 

Q. 11 According to Piaget theory, which one out of the following will not influence one’s cognitive development?

A. Social experiences

B. Maturation

C. Activity

D. Language

 

Q. 12 Which of these does not imply practical intelligence in the Triarchic theory?

A. Thinking practically about oneself only

B. Choosing an environment in which you can succeed

C. Adapting to the environment

D. Reshaping the environment

 

Q. 13 “Anyone can become angry – that is easy, but to be angry with the right person, to the right degree, at the right time, for the right purpose, and in the right way – that is not easy.” This is related to

A. Social development

B. Cognitive development

C. Physical development

D. Emotional development

 

Q. 14 In learning, assessment is essential for

A. Screening test

B. Motivation

C. Fostering of the purpose of segregation and ranking

D. Grades and Marks

 

Q. 15 Fitting new information into existing schemes is known as

A. Equilibrium

B. Assimilation

C. Organisation

D. Accomodation

 

Q. 16 We all differ in terms of our intelligence, motivation, interest, etc. This principle refers to 

A. Theories of Intelligence

B. Heredity

C. Environment

D. Individual differences

 

Q. 17 Students of disadvantaged groups should be taught along with the normal students. It implies

A. Special Education

B. Integrated Education

C. Exclusive Education

D. Inclusive Education

 

Q. 18 ‘Out-of-the-box’ thinking is related to

A. Memory based Thinking

B. Divergent Thinking

C. Convergent Thinking

D. Consistent Thinking

 

Q. 19 The assessment of students can be used by teachers in teaching to develop insight into

A. not promoting those students who do not meet school standards

B. changing the teaching approach according to the learners’ need

C. creating groups of ‘bright’ and ‘weak’ students in the class

D. identifying the students who need to be promoted to the higher class

 

Q. 20 Learning experiences should be planned in a manner so as to make learning meaningful. Which of the given learning experiences does not facilitate meaningful learning for the children?

A. Formulating questions on concept

B. Discussion and debate on the topic

C. Presentation on the topic

D. Repetition based on mere recall of content

 

Q. 21 Giving punishment, verbal or non-verbal, to the children results in

A. protecting the child’s image

B. improving their scores

C. damaging their self-concept

D. motivating them to work

 

Q. 22 A teacher, labelled the head of a committee, as ‘chairperson’ instead of ‘chairman’. It indicates that the teacher

A. has a good command of language

B. is using a gender-free language

C. has gender bias

D. follows a more acceptable term

 

Q. 23 Continuous and Comprehensive Evaluation is essential for

A. diluting the accountability of the Board of Education

B. correcting less-frequent errors more than more-frequent errors

C. understanding how learning can be observed, recorded and improved upon

D. fine tuning of test with the teaching

 

Q. 24 In Lawrence Kohlberg’s theory, which level signifies the absence of morality in the true sense?

A. Level IV

B. Level I

C. Level II

D. Level III

 

Q. 25 Which one of the following is not correct for the progressive model of socialisation of children?

A. Children accept what they are offered by the school irrespective of their social backgrounds

B. They should be a place for democracy in the classroom

C. Socialisation is an adoption of social norms

D. Active participation in the group work and learning social skills

 

Q. 26 The teacher noticed that Pushpa cannot solve problem on her own. However, she does so in the presence of adult or peer guidance. This guidance is called

A. Pre-operational thinking

B. Zone of proximal development

C. Scaffolding

D. Lateralization

 

Q. 27 Which one out of the following provides information about the roles and behaviours which are acceptable in a group, during early childhood period?

A. Teachers and Peers

B. Peers and Parents

C. Parents and Siblings

D. Siblings and Teachers

 

Q. 28 Which of the following age groups falls under later childhood category?

A. 18 to 24 years

B. Birth to 6 years

C. 6 to 11 years

D. 11 to 18 years

 

Q. 29 Aarjav says that language development is influenced by one’s innate predisposition while Sonali feels that it is because of the environment. This discussion between Aarjav and Sonali is about

A. Stability and Instability argument

B. Continuous and Discontinuous learning

C. Nature and Nurture debate

D. Critical and Sensitive feeling

 

Q. 30 Making students members of a cleanliness community to motivate them for the same, reflects

A. Behaviouristic approach to motivation

B. Humanistic approach to motivation

C. Cognitive approach to motivation

D. Socio-cultural conceptions to motivation

 

Q. 31 The sum of place values of 5 in 6251, 6521 and 5621 is

A. 15

B. 5550

C. 5050

D. 550

 

Q. 32 Which one of the following statements is true?

A. The difference of an even number and an odd number can be an even number.

B. The sum of two odd numbers and one even number is an even number.

C. The sum of three odd numbers is an even number.

D. The product of three odd numbers is an even number.

 

Q. 33 A one-litre carton of juice is in the shape of a cuboid and has a square base of size 8 cm by 8 cm. The depth of juice in the carton, in centimeters, is closest to

A. 22

B. 16

C. 18

D. 20

 

Q. 34 Which one of the following does not match curricular expectations of teaching

mathematics at the primary level?

A. Analyse and infer from representation of grouped data

B. Develop a connection between the logical functioning of daily life and that of mathematical thinking

C. Develop language and symbolic notations with standard algorithms of performing number operations

D. Represent part of whole as a fraction and order simple fractions

 

Q. 35 The main goal of Mathematics education is

A. to help the students to understand mathematics.

B. to develop useful capabilities

C. to develop children’s abilities for mathematization

D. to formulate theorems of Geometry and their proofs independently.

 

Q. 36 At primary level use of tangram, dot games, patterns, etc. helps the students to

A. enhance spatial understanding ability

B. develop sense of comparing numbers

C. strengthen calculation skills

D. understand basic operations

 

Q. 37 From the unit if ‘Shapes’ the teacher asked the students to “Make/draw and picture by using shapes”.

The objective that can be achieved through this activity is

A. Knowledge

B. Comprehension

C. Creating

D. Application

 

Q. 38 Arjun, a student of class IV, is able to answer all questions related to Number System orally, but commits mistakes while writing the solutions of problems based on Number System. The best remedial strategy to remove errors in his writing is

A. to relate real life experiences with mathematical concepts

B. to provide him a worksheet with partially solved problems to complete the missing gaps

C. to teach more than one way of solving problems of Number System

D. to give him 10 practice tests

 

Q. 39 What is the value of

-1 + 2 – 3 + 4 – 5 + 6 – 7 + … + 1000?

A. 2000

B. 0

C. 1

D. 500

 

Q. 40 Perimeter of a square is 44 cm. The perimeter of a rectangle is equal to the perimeter of this square. The length of the rectangle is 5 cm more than the side of the square. The sum of areas (in cm²) of the square and the rectangle is

A. 229

B. 169

C. 140

D. 217

 

Q. 41 As per the NCF 2005, the narrow aim of teaching Mathematics at schools is

A. to develop numeracy related skills

B. to teach algebra

C. to teach calculation and measurements

D. to teach daily life problems related to linear algebra

 

Q. 42 Ravi has three dozen chocolates. He gave one-third of them to his neighbour, one-sixth to Rehana and one-fourth to his sister. How many chocolates are left with him?

A. 10

B. 6

C. 8

D. 9

 

Q. 43 A child who is able to perform all number operations and is able to explain the concept of fractions is at

A. operational phase

B. emergent phase

C. quantifying phase

D. partition phase

 

Q. 44 A teacher introduced multiplication in her class as repeated addition and then by grouping of same number of objects taken multiple times she introduced the ‘x’ symbol and further conducted a small activity of finding product using criss-cross lines or matchsticks. Here the teacher is

A. using multiple representations to make the class interesting.

B. developing a lesson and taking students ‘from concrete to abstract concept’.

C. catering to learners with different learning styles.

D. providing remedial strategies for low achievers in mathematics.

 

Q. 45 Akansha wants to become good mathematics teacher. To be a good mathematics teacher she must have

A. a

B. b

C. c

D. d

 

Q. 46 It is important to conduct mathematical recreational activities and challenging geometrical puzzles in the class as

A. they bring students out of the monotonous and boring routines of mathematical classroom.

B. they give space to gifted learners.

C. they are helpful to enhance spatial and analytical ability of every learner.

D. they can create interest in low achievers and slow learners in mathematics.

 

Q. 47 ‘Vedic Mathematics’ is becoming popular nowadays especially amongst primary school children and is used to enhance

A. the problem solving skills of students in mathematics

B. the concentration of students in mathematics

C. the calculation skills and speed in mathematics

D. the algorithmic understanding of students in mathematics

 

Q. 48 Formative Assessment in Mathematics at primary stage includes

A. identification of common errors

B. testing of procedural knowledge and analytical abilities

C. grading and ranking of students

D. identification of learning gaps and deficiency in teaching

 

Q. 49 A teacher uses the exploratory approach, use manipulatives and involvement of students in discussion while giving the concepts of mathematics. She uses this strategy to 

A. develop manipulative skills among the students

B. create a certain way of thinking and reasoning

C. achieve the narrow aim of teaching mathematics

D. achieve the higher aim of teaching mathematics

 

Q. 50 A teacher asks Shailja of Class V about the perimeter of a figure.

She also asked Shailja to explain the solution in her words. Shailja was able to solve the problem correctly but was not able to explain it. This reflects that Shailja is having

A. poor understanding of concept of perimeter but good verbal ability

B. lower language proficiency and lower order mathematical proficiency

C. lower language proficiency and higher order mathematical proficiency

D. poor confidence level and poor mathematical skills

 

Q. 51 The section, ‘Practice Time’ included in different topics in Mathematics textbook aims at 

A. having a change in daily routine

B. ensuring better utilization of time

C. providing extended learning opportunities

D. providing fun and enjoyment to students

 

Q. 52  13 students of class V A and 15 of Class V B participated in a writing competition. They scored marks as follows:

Class V A : 14, 6, 15, 12, 11, 11, 7, 9, 17, 13, 3, 10, 18

Class V B : 13, 9, 0, 7, 14, 6, 0, 9, 16, 9, 13, 16, 5, 18, 11

What inference can you draw from the given data?

A. Class V A performed better because the average score of V A is more

B. Both the sections performed equally well because the total marks scored by both the sections are the same

C. Both the sections performed equally well because the average marks of both the sections are the same

D. Both the sections performed equally well because the highest score of both the sections is 18

 

Q. 53 How many 1/10 are in 6/5?

A. 5

B. 12

C. 10

D. 8

 

Q. 54 On dividing 110111 by 11, the sum of the quotient and the remainder is

A. 11001

B. 10101

C. 10011

D. 11011

 

Q. 55 What should be subtracted from the product 102 x 201 to get 19999?

A. 602

B. 103

C. 401

D. 503

 

Q. 56 Which of the following is not correct?

A. 2 kg 30 g is the same as 2030 g

B. 3 litres 80 millilitres = 380 millilitres

C. Area of a square of side 10 cm = Area of the rectangle of length 100 cm and breadth 0.01 m

D. 3 hours 14 minutes = 194 minutes

 

Q. 57 Number of degrees in two and two-third of a right-angle is

A. 270

B. 180

C. 210

D. 240

 

Q. 58 (Sum of multiples of 7 between 21 and 49) ÷ (Biggest common factor of 25 and 30) is equal to

A. 37

B. 14

C. 21

D. 35

 

Q. 59 The sum of all the positive factors of 96 is

A. 252

B. 155

C. 156

D. 251

 

Q. 60 A train leaves a station at 6 : 14 a.m. and reaches its destination after 13 hours 48 minutes. The time at the destination is

A. 8 : 12 p.m.

B. 7 : 02 p.m.

C. 7 : 12 p.m.

D. 8 : 02 p.m.

 

Q. 61 NCF 2005 has not recommended any prescribed curriculum and textbooks for EVS for classes I and II. The most appropriate reason for this:

A. EVS is only for class III onwards

B. Learners in Class I and II cannot read and write

C. To provide contextual learning environment

D. To reduce the load of curriculum

 

Q. 62 A teacher has to teach about cultural diversity in food in our country to class IV students. Which of the following is the best way to teach the topic?

A. Give a project to students about different kinds of food taken by the people of various states in our country

B. Ask students what they have eaten, followed by a discussion

C. Ask students to collect information about the kind of food taken in their family

D. Show flash cards having pictures of various food items

 

Q. 63 All of the following are indicative of the general approach to be followed for EVS in primary classes except from

A. concrete to abstract

B. simple to complex

C. local to global

D. abstract to concrete

 

A. Only D

B. Only A and B

C. Only B and C

D. Only B

 

Q. 64 Study the table given in the figure.

After studying the table, Devmeet a class V student, wrote the following in his notebook:

A. Petrol is a more expensive fuel than diesel.

B. The percentage increase in price of diesel is more than that of petrol.

C. Diesel is cheaper and that is why public transport runs on it.

D. Diesel is a cheaper fuel than petrol. People should buy diesel vehicles.

Which one of the following is correct?

A. C and D are inferences

B. A and D are observations

C. A and C are observations

D. B and C are inferences

 

Q. 65 Mahika, a teacher of Class V, wants her students to observe the plants in the surroundings. For meaningful learning, what should she encourage the children to do?

A. Appreciating that plants are useful for us

B. Writing the names of maximum number of plants

C. Observing the differences in their heights, leaves, smell and place of growing

D. Understanding that plants are living beings

 

Q. 66 Which of the following should not be an appropriate indicator for assessment in EVS at primary level?

A. Questioning

B. Concern for justice and equality

C. Cooperation

D. Remembering

 

Q. 67 Your principal asks you to take class V students on an educational visit. What would you include while planning for such a visit to make it meaningful?

A. Asking students to find and read the information about the place to be visited

B. Taking enough food and games to enjoy

C. Ensuring that students should be disciplined and should visit the place quietly

D. Preparing structured activity sheet about the place to be visited and sharing the same with the students before the visit for an authentic and meaningful learning

 

Q. 68 Stories and poems can be effectively used in teaching of EVS at primary level. The main purpose of using them in EVS is to

A. providing contextual leaning environment

B. make lessons enjoyable

C. promote linguistic skills

D. cater to diversity in classroom

 

Q. 69 How would a primary teacher effectively assess her students of class IV on the topic of “Scarcity of Water”?

A. Conducting a written test on the above topic

B. By finding out how they have started saving water in their day-to-day life

C. Organising a poster making activity on water conservation

D. Asking children to write slogans on saving water

 

Q. 70 ‘Mapping’ at primary level promotes which of the following skills in learners?

A. Calculations and Estimations

B. Drawing according to scale

C. Idea about relative positions and orientations

D. Neat drawing

 

Q. 71 This plant has leaves which are used as vegetables. Its seeds are used to produce oil. The plant is

A. Coconut

B. Cabbage

C. Mustard

D. Spinach

 

Q. 72 Which one of the following is a set of diseases caused/spread by mosquitoes?

A. Cholera, Dengue, Malaria

B. Chikungunya, Dengue, Malaria

C. Chikungunya, Cholera, Typhoid

D. Chikungunya, Malaria, Typhoid

 

Q. 73 On a state map, at its one corner it is mentioned ‘Scale 1 cm = 110 metres”. If on the map the measured distance between any two cities is 9.7 cm, the actual distance between the two cities is approximately

A. 2.01 km

B. 11 km

C. 10 km

D. 1.067 km

 

Q. 74 Study the following statements about bronze:

A. It is an element like aluminium, iron and copper.

B. It is made by melting copper and tin in a furnace.

C. Tribal people have been using bronze since thousands of years.

D. The objects made of bronze are lighter but much stronger than those made of aluminium.

The correct statements are

 

A. Only B and C

B. A, B and C

C. B, C and D

D. Only A and D

 

Q. 75 Which of the following is/are tools and techniques of assessment in EVS at primary level?

A. Project work.

B. Field trip.

C. Journal writing

D. Concept mapping

 

A. Only D

B. A, B, C and D

C. Only A and B

D. Only B and C

 

Q. 76 Which one of the following strategies seems most appropriate for teaching maps to primary graders?

A. Instructing learners to bring the map of India from their home

B. Showing an atlas to learners and asking them to locate different place

C. Facilitating learners to construct maps of their immediate surroundings by using their own symbols and focusing on relative position and direction of things

D. Drawing of the map by the teacher on the black-board and asking learners to locate different places

 

Q. 77 Which one of the following should be a most important aspect of EVS teaching at primary level?

A. Achieving success in assessment

B. Understanding the basic concepts of science

C. Connecting learners to the natural and socio-cultural environment

D. Conducting activities and developing skills

 

Q. 78 Teaching of EVS should encourage process skills, which are the core of inquiry-based, hands-on learning. Which one of the following is not such a skill?

A. Determination

B. Inferring

C. Observation

D. Prediction

 

Q. 79 Study the following statements about the Jhoom farming practised in Mizoram:

A. After cutting one crop, the land is left as it is for some years.

B. The bamboo or weeds which grow on that land are cut and burnt.

C. The land is deeply ploughed before sowing the seeds.

D. In one farm three or four different types of crops are grown.

E. Chemical fertilizers and pesticides are also used as per need.

The correct statements are

 

A. A, B and D

B. Only A and B

C. Only B and D

D. C, D and E

 

Q. 80 Radha uses travel tickets of different types. Which of the following concepts could be effectively discussed by using such tickets?

A. The cost of travel by different means is different.

B. The cost of travel depends upon the fuel used by the vehicle.

C. We need tickets to travel by public transport from one place to another.

D/. Understanding the reservation process.

Choose the most appropriate option:

 

A. A and D

B. A and C

C. B and C

D. C and D

 

Q. 81 If we observe birds, we find that most of the birds often move their neck. It is because 

A. their ears are covered and they can fly

B. the birds have two eyes

C. their eyes can focus on two different objects at a time

D. in most of the birds, the eyes are fixed and cannot move

 

Q. 82 Reena loves mangoes. She wants to preserve these for winters. Which one of the following is a good way for preserving them?

A. Prepare juice and store in an airtight container.

B. Put in a plastic bag.

C. Store in a refrigerator.

D. Make ‘aam papad’ and pickle.

 

Q. 83 Which one of the following animals’ front teeth keep on growing throughout its life? It has to keep gnawing on things to keep its teeth from becoming too long.

A. Squirrel

B. Lizard

C. Rat

D. Termite

 

Q. 84 A farmer is growing paddy crop over and over again using excess of fertilizers and pesticides in his fields. This practice will make the soil of this fields

A. fit for ploughing

B. barren

C. fertile for other crops also

D. useful for the paddy crops only

 

Q. 85 The neighbouring states of Kerala are

A. Karnataka and Maharashtra

B. Andhra Pradesh and Karnataka

C. Karnataka and Tamil Nadu

D. Tamil Nadu and Andhra Pradesh

 

Q. 86 In which one of the following states of our country do most people like to eat sea fish cooked in coconut oil?

A. Goa

B. Jammu and Kashmir

C. Bihar

D. Mizoram

 

Q. 87 Study the following statements about “Desert Oak”.

A. Desert Oak is a tree that grows almost as tall as our classrooms, i.e., about 4 meters.

B. The roots of this tree go down nearly 30 times its height till they reach water.

C. Water is stored in the trunk of this tree.

D. This tree is found in Abu Dhabi.

The correct statements are

 

A. B, C and D

B. Only A and B

C. A, B and C

D. A, B and D

 

Q. 88 A person boarded a train on 02.01.2015 at Madgaon for Nagercoil. The train departed from Madgaon at 07 : 45 hours and reached Nagercoil at 04 : 45 hours onthe next day i.e., 03.01.2015. If the distance between these two stations is 1140 km, the average speed of the train was approximately

A. 51 km/h

B. 59 km/h

C. 57 km/h

D. 54 km/h

 

Q. 89 A person living in New Delhi wants to visit first Bhopal (Madhya Pradesh) and then Ranchi (Jharkhand). The directions of his journey will be first towards

A. west and then towards south

B. south and then towards east

C. south and then towards west

D. east and then towards south

 

Q. 90 Which one of the following characteristics is shared by all the animals given below? 

Lizard, Sparrow, Turtle, Snake

A. They are poisonous

B. They can live on land as well as in water

C. Their bodies are covered with scales

D. They lay eggs

 

Q. 91 Role Play should be an integral part of every language classroom because

A. it is an effective classroom management technique

B. it enables students to engage in meaningful talk

C. it enables students to memorize the story

D. it enables girls and boys to interact freely

 

Q. 92 A language teacher wants her students to write for an authentic audience and purpose. What would be the best writing task to achieve this?

A. Students write answers to questions given at the end of the chapter after discussing them with each other.

B. Students write a movie review of a movie that watched recently.

C. Students write on the topic, “My School”.

D. Students write a letter to the principal expressing their opinion on a school-related issue.

 

Q. 93 Which one of the following is an example of Basic Interpersonal Communication Skills (BICS)?

A. Discussing response to a story in a small group in the classroom

B. Taking notes during a lecture or a talk

C. Negotiating turn taking with a peer during free play

D. Explaining a procedure to a peer during an experiment

 

Q. 94 B. F. Skinner claimed that language is learnt through

A. reinforcement and engagement

B. drill and practice

C. immersion and employment

D. repetition and approximation

 

Q. 95 Rani is from Delhi. She has taken admission in a school in Assam. In the final exams, she faces difficulty in writing a composition on ‘Bihu’. The most probable reason for Rani’s problem is that

A. Her parents are unable to help her in understanding the school culture.

B. She is not a hardworking girl and is not trying to adapt to the school curriculum.

C. Her background is deficient and is not on a par with the standard school culture.

D. There is a discontinuity between her home environment and the school curriculum.

 

Q. 96 A language textbook for class 1 starts with poems and stories (complete text) and ends with alphabets. Which approach does this kind of arrangement reflects in language pedagogy?

A. Top-down approach

B. Aesthetic approach

C. Bottom-up approach

D. Eclectic approach

 

Q. 97 Which of the following is not true about sentences in the passive voice?

A. The verb in passive sentences is always in perfect participle form.

B. It is generally used where the subject is hidden, not clear or not significant.

C. Sentences with intransitive verbs cannot be passivized.

D. Any declarative sentence can be passivized.

 

Q. 98 A child reads ‘She bought three apples’ as ‘She bought tree apples’ and explains it as ‘apples from trees’. How would you rate this child’s reading skills?

A. She reads with spelling errors.

B. She reads carelessly.

C. She reads with comprehension.

D. She reads without comprehension.

 

Q. 99 The most important purpose of formative assessment is

A. to form an opinion about the best student in the class

B. to score and rank students on the basis of their performance

C. to provide qualitative feedback on students’ learning

D. to judge the performance of teachers and schools

 

Q. 100 In the primary classes, it is recommended that children should be taught in their mother tongue because

A. it creates a natural environment

B. children develop self-confidence

C. it promotes regional languages

D. it enables children to comprehend

 

Q. 101 A student of class V has just read a text about the water problems of a community living on the outskirts of a city. After reading the chapter, the student remarks how difficult it must be to struggle for water every day. Which comprehension strategy does this remark reveal? 

A. Summarizing

B. Metacognition

C. Inferential reading

D. Read aloud

 

Q. 102 A common developmental reading disorder is

A. Dyslexia

B. Aphasia

C. Autism

D. Impairment

 

Q. 103 Writing is a _____________ and not a ____________.

A. product; formation

B. process; product

C. process; formation

D. product; process

 

Q. 104 Which one of the following principles is not appropriate for vocabulary development? 

A. Provide opportunities to consult a dictionary

B. Integrate new words with old

C. Make a new word ‘real’ by connecting it to the learners’ world

D. Passive vocabulary should be learnt by heart

 

Q. 105 Storytelling should be used frequently in classrooms because 

A. it lays the foundation of logical understanding and imagination

B. it lays the foundation for other academic engagements

C. it allows students to imagine and relax

D. it provides space for teachers to engage in other academic tasks

 

Questions: 106 – 114

Read the passage given below and answer the questions that follow:

Adversity provides us with an opportunity to develop our character in a natural, recurring and powerful way that only the challenges of adversity offer. According to Solomon, adversity refines and reveals the gold and silver of our character. A lot of times adversity comes our way as a direct or indirect result of our actions. We make a bad choice or a bad decision, or we simply fail to do something we should have done. When I made bad investment decisions, I had to accept responsibility for my greed and naive choices. Yes, several men had misrepresented the opportunities to me, but the fact is, I am the one who made the decisions. And I experienced the very consequences, that Solomon had cautioned us about. Any time you make a contribution to your own adversity, you need accept responsibility for it. Don’t simply blame someone or something else. Nonetheless, throughout our lives we will experience a great deal of adversity that is not a result of our own actions. It is critically important that we do not assign fault to ourselves or to those who had nothing to do with it. When a friend of mine lost his daughter to leukemia, he confided to me that he felt God was punishing him for his past sins. In other words, he was blaming himself. It is believed that adversity sometimes has a purpose that we cannot know or understand. As tempting as it may be, to try to figure out such a mystery is not only an exercise in futility, it is foolish also.

 

Q. 106 Adversity provides us with an opportunity to

A. develop our character

B. test our friends

C. evaluate our own character

D. introspect

 

Q. 107 The author quotes Solomon to

A. show his veneration for him

B. emphasize that adversity is part of life

C. embellish his prose

D. lend force to his argument

 

Q. 108 Most often our misfortunes are the result of our own

A. haste

B. follies

C. actions

D. idleness

 

Q. 109 The synonym for ‘cautioned’ is

A. threatened

B. suggested

C. persuaded

D. warned

 

Q. 110 The phrase ‘exercise in futility’ means

A. something that is pointless

B. hopes of future

C. an irrational act

D. a foolish approach

 

Q. 111 Identify the correct statement.

A. Adversity is a curse

B. The mystery of adversity can be easily understood

C. Adversity helps us improve our character

D. Adversity is purposeless

 

Q. 112 Which of the following statements is not true?

A. The bravest are bogged down by misfortunes

B. Adversity refines our character

C. Adversity sometimes has a purpose

D. Adversity is a test of our character

 

Q. 113 The antonym for the word ‘adversity’ is

A. luxury

B. luck

C. emptiness

D. prosperity

 

Q. 114 When adversity strikes us we blame

A. providence

B. our stars

C. everything and everyone except ourselves

D. supernatural powers and evil spirits

 

Questions: 115 – 120

Read the poem given below and answer the questions that follow:

Invictus

Out of the night that covers me,

Black as the pit from pole to pole,

I thank whatever gods may be

For my unconquerable soul.

In the fell clutch of circumstances

I have not winced nor cried aloud.

Under the bludgeoning of chance

My head is bloody, but unbowed.

Beyond this place of wrath and tears

Looms but the Horror of the shade,

And yet the menace of the years

Finds, and shall find, me unafraid.

William Ernest Henley

 

Q. 115 The phrase ‘unconquerable soul’ means a person who is

A. compassionate

B. noble

C. sensitive

D. invincible

 

Q. 116 Lines 5 and 6 show that the speaker

A. remains undaunted even under the worst circumstances

B. is overwhelmed by adverse circumstances

C. accepts life’s challenges

D. refuses to surrender

 

Q. 117 ‘Wrath and tears’ means

A. anger causing havoc

B. anger and sorrow

C. unfavourable circumstances

D. unbearable suffering

 

Q. 118 The phrase ‘menace of the years’ suggests

A. danger to life

B. cruel fate

C. evils of life

D. threats of the times

 

Q. 119 The word ‘winced’ in the second stanza means

A. ruffled

B. frightened

C. worried

D. recoiled

 

Q. 120 The poetic device used in ‘Black as the pit from pole to pole’ is

A. irony

B. simile

C. parallelism

D. metaphor

 

Questions: 121 – 126

Read the passage given below and answer the questions that follow:

As District Employment Officer, my Father was given a jeep by the government. There was no garage in the office, so the jeep was parked in our house. My father refused to use it to commute to the office. He told us that the jeep is an expensive resource given by the government – he reiterated to us that it was not ‘his jeep’ but the government’s jeep. Insisting that he would use it only to tour the interiors, he would walk to his office on normal days. He also made sure that we never sat in the government jeep – we could sit in it only when it was stationary. That was our childhood lesson in governance – a lesson that corporate managers learn the hard way, some never do. The driver of the jeep was treated with respect due to any other member of my Father’ office. As small children, we were taught not to call him by his name. We had to use the suffix ‘dada’ whenever we were to refer to him in public or private. When I grew up to own a car and a driver by the name of Raju was appointed, I repeated the lesson to my two small daughters. They have, as a result, grown up calling him ‘Raju Uncle’ – very different from many of their friends who refer to their family drivers as ‘my driver’. When I hear that term from a school or college going person, I cringe. To me, the lesson was significant – you treat small people with more respect than you treat big people. It is more important to respect your subordinates than your superiors.

 

Q. 121 The author’s father would not allow his family to use the jeep because

A. the roads were full of potholes

B. he was afraid of accidents

C. the jeep was in a bad conditions

D. it was not their private vehicle

 

Q. 122 The author taught his children to

A. treat small people with respect

B. maintain a discreet distance from servants

C. be kind to small people

D. be firm with servants

 

Q. 123 The author was critical of his children’s friends because their attitude to servants smacked of

A. coarseness

B. arrogance

C. loftiness

D. weakness

 

Q. 124 The author’s attitude towards servants can be described as

A. indifferent

B. rational

C. affectionate

D. respectful

 

Q. 125 The opposite of the word refused’ is

A. received

B. justified

C. admired

D. accepted

 

Q. 126 The word that can replace ‘reiterated’ is

A. repeated

B. recalled

C. reconsidered

D. revised

 

Questions: 127 – 135

Read the passage given below and answer the questions that follow:

Srinivasa Ramanujan (1887 – 1920)

Ramanujan was born on December 2, 1887, in Erode (South India) as the eldest son in a family of six children. In November 1892, he entered, the Town High School at Kumbakonam as a half-fee scholarship-holder and passed the Matriculation Examination in 1904. In the school, he became a minor celebrity, walking off with merit certificates and prizes for academic brilliance. This school nourished him for six years, bringing him as close as he would ever come to a satisfying academic experience. When he was in the seventh standard, he gave clear evidence of his mathematical gifts; he could reel off the square root of a natural number to the specified number of places; he could point to the indeterminate nature of zero divided by zero. Ramanujan’s mother – the family being close to penury – took in college students as boarders who noticing Ramanujan’s interest in mathematics, bought him textbooks from the college library. Loney’s ‘Trigonometry’ was one such treasure which be mastered. During 1906-1912, Ramanujan was constantly in search of an employer to earn his livelihood. With his ‘Note-books’ as his only recommendation, he sought the patronage of V. Ramaswamy Iyer, the founder of Indian Mathematical Society who was at Tirukovillur and asked for a clerical job in his office. The former had no mind to smother Ramanujan’s genius and sent him back to Madras with a letter of introduction to P. V. Seshu Aiyar, then at the Presidency College, Madras. He gave in turn, Ramanujan a letter of recommendation to that true lover of mathematics, R. Ramachandra Rao, the District Collector, Nellore. This was the turning point in his life. On the advice of P. V. Seshu Aiyar, Ramanujan communicated his theorems on divergent series in a historic letter dated January 16, 1913 to G. H. Hardy, who was ten years senior to Ramanujan. With the personal interest of Gilbert Walker and support given by Indian stal.. , then University of Madras awarded its first scholarship to Ramanujan to study in Cambridge. Over the next three months, Ramanujan received four long letters from Hardy, who had already sprung into action, advising the India Office, of his wish to bring him to Cambridge. 

 

Q. 127 Merit certificates and prizes awarded to Ramanujan at school are a proof of his

A. intellectual brilliance

B. sincerity

C. dedication

D. commitment

 

Q. 128 Ramanujan’s mother took in college students as boarders because

A. she wanted to give her son all the comforts of life

B. she wanted to save money to buy a house

C. she had to pay up huge debts

D. the family was on the verge of property

 

Q. 129 The turning point in Ramanujan’s life came when

A. he got a job in Indian Mathematical Society

B. his name was recommended to the District Collector, Nellore

C. he was awarded a big cash award

D. he was given a scholarship

 

Q. 130 Identify the correct statement.

A. Ramanujan did not get much support from his school

B. Ramanujan’s mother did not want him to go abroad

C. Seshu Aiyar was Ramanujan’s patron

D. Ramanujan was offered a job at Cambridge

 

Q. 131 The phrasal verb, ‘reel off’ means to

A. fishing

B. rehearse easily

C. articulate fast

D. say quickly

 

Q. 132 The closest synonym for the word ‘smother’ is

A. stifle

B. discourage

C. ruin

D. deaden

 

Q. 133 The antonym for the word ‘recommendation’ is

A. condemnation

B. criticism

C. revulsion

D. disapproval

 

Q. 134 The word that can best replace ‘nourished’ is

A. sent

B. supported

C. served

D. gifted

 

Q. 135 To support Ramanujan received from his school suggests that

A. a talented person needs nourishment to flourish

B. luck is more important than patronage

C. support or no support, men with talent forge ahead

D. fortune favours those who dares

 

Q. 136 Skilled reading is

A. Deliberate

B. Constructive

C. Imaginative

D. Progressive

 

Q. 137 Of the following, which one is the most important prerequisite for language learning, whether first or second?

A. Skills-based instruction

B. A multilingual approach

C. An input-rich communicational environment

D. A structural-situational approach

 

Q. 138 A Hindi-speaking teacher gets posted in a primary school which is situated in a remote area of Rajasthan. Since she doesn’t know the local language, she faces lots of problems. She should

A. focus on the textbook as source of standard Hindi

B. use the child’s language as a resource while teaching

C. encourage the community to learn standard Hindi

D. try to get a posting to a Hindi-speaking area

 

Q. 139 While teaching children to read, at which point should the teacher focus on

comprehension?

A. After children have learned how to decode

B. Right from the beginning

C. Once children have mastered phonics

D. When children reach class II

 

Q. 140 Here is a list of tasks commonly included in a language classroom. Which of these sees children as active learners?

A. Children carefully memorise correct answers to questions on a poem

B. Children write answers to questions given at the end of a poem

C. Children carefully note down answers from the blackboard

D. Children work in groups to generate interpretations of a poem

 

Q. 141 The assessment of students’ writing should most importantly focus on

A. using idioms and metaphors

B. correct spelling and grammar

C. expressions and ideas

D. keeping to the world limit

 

Q. 142 Choose the correct spelling.

A. Dosent

B. Doesn’t

C. Does’nt

D. Doesnt’

 

Q. 143 A primary teacher should introduce reading through

A. picture books

B. alphabet books only

C. phonic teaching

D. stories

 

Q. 144 A teacher of class III realizes that vocabulary development is an important factor in enabling students to become better readers. Of the following, which might be a good strategy for vocabulary development?

A. Students learn to use the context to guess the meaning of new words

B. Students memorise extensive word-lists of synonyms and antonyms

C. Students consult a dictionary whenever they come across a new word

D. Students underline difficult words from a text and make sentences with them

 

Q. 145 Children’s oral language development forms an important foundation for learning literacy. Which of the following classroom practices enables oral language development?

A. Chorus reading of a story in the textbook along with the teacher

B. Practising the correct pronunciation of words in a chorus

C. Participating in role-plays on favourite stories

D. Memorising and reciting poems individually or in a chorus

 

Q. 146 Which one of the following would be the best evidence to demonstrate to parents and administrators what students can do with language?

A. Lists of course goals and objectives

B. Marks in a test

C. Poems or paragraphs written by students

D. National curriculum and syllabi

 

Q. 147 Before students start reading a story titled, ‘Brave Bitto’, the teacher initiates a discussion with them on ‘Bravery’. What is the teacher trying to achieve through this activity?

A. Activate the intellectual stance of students

B. Activate enthusiasm in students

C. Activate the efferent stance in students

D. Activate the previous knowledge of students

 

Q. 148 Mrs. Sinha asks prediction questions as she reads aloud a story to her class III students. She does this to

A. make the story interesting

B. focus in comprehension

C. help students remember important details in the story

D. improve students’ vocabulary

 

Q. 149 Scribbling is a stage of

A. Listening

B. Writing

C. Reading

D. Speaking

 

Q. 150 If a student is making pronunciation errors, the best way to help him/her is to

A. call his/her parents and complain

B. scold him/her in class for incorrect pronunciation

C. mock at him/her in class for incorrect pronunciation

D. provide him/her with correct pronunciation without any humiliation

 

Answer Sheet 
Question123456 78910
AnswerBDBADAACCD
Question11121314151617181920
AnswerAADBBDDBBD
Question21222324252627282930
AnswerCBCBACCCCD
Question31323334353637383940
AnswerCBBACACBDD
Question41424344454647484950
AnswerADABACCDBC
Question51525354555657585960
AnswerCABCDBDCAD
Question61626364656667686970
AnswerCAAACDDABC
Question71727374757677787980
AnswerCBDABCCAAB
Question81828384858687888990
AnswerDDABCACDBD
Question919293949596979899100
AnswerBDCBDADDCD
Question 101102103104105106107108109110
AnswerCABDAADCDA
Question 111112113114115116117118119120
AnswerCADCDABDDB
Question121122123124125126127128129130
AnswerDABDDAADBC
Question131132133134135136137138139140
AnswerDADBABCBBD
Question141142143144145146147148149150
AnswerCBAACCDBBD

CTET February 2014 Paper-I Previous Year Paper

CTET February 2014 Paper-I

Q. 1 In the progressive model of education as implemented by CBSE, socialization of children is done in such a way so as to expect them to

A. give up time consuming social habits and learn how to score good grades

B. be an active participant in the group work and learn social skills

C. prepare themselves to conform to the rules and regulations of society without questioning

D. accept what they are offered by the school irrespective of their social background

 

Q. 2 which of the following is based on vygotsky’s sociocultural theory?

A. operant conditioning

B. reciprocal teaching

C. cultural neutral cognitive development

D. insight learning

 

Q. 3 A teacher says to her class, ”As individual assignments are designs to help individual students learn more effectively, a;; students should complete assignments prescribed without any assistance”. she is referring to which of the following stages of kohlberg’s moral development.

A. conventional stage 4-law and order

B. post conventional stage 5 –social contract

C. pre conventional stage 1–punishment avoidance

D. pre conventional stage 2 –individualism and exchange

 

Q. 4 fourteen-year old girl devika attempting to develop a sense of herself as a separate, self governing individual. she is developing.

A. hatred for rules

B. autonomy

C. teenage arrogance

D. maturity

 

Q. 5 In context of progressive education, which of the following statements is true according to

John Dewey

A. there should not be a place for democracy in a classroom

B. students should be able to solve social problems themselves

C. curiosity does not belong to the inherent nature of the students rather it is to be cultivated

D. students should be observed and not heard in the classroom

 

Q. 6 A disorder related to the language comprehension is

A. apraxia

B. sydlexia

C. aspeechxia

D. aphasia

 

Q. 7 following are the critical views about the “theory of the multiple intelligences,” except 

A. it is not research based

B. different intelligences demand different methods for different students

C. gifted students usually excel in a single domain

D. it lacks of empirical support

 

Q. 8 theory of multiple intelligences cannot be legitimized as it

A. is not possible to measure different intelligences as there are no specific tests

B. does not place equal importance on all seven intelligences

C. is based on the sound empirical studies done by abraham maslow throughout his life

D. is not compatible with general intelligence “g”, which is most important

 

Q. 9 The individual differences of students in a classroom are

A. disadvantages as teachers need to control a diverse classroom

B. detrimental as they lead to student-student conflicts

C. inexpedient as the reduce the speed of the curriculum transaction to the level of the slowest student

D. advantageous as they lead teacher to explore a wider pool of cognitive structures

 

Q. 10 school-based assessment was introduced to

A. decentralize the power of boards of school education in the country

B. ensure the holistic development of all the students

C. motivate teachers to punctiliously record all the activities of students for better interpretation of their progress

D. encourage schools to excel by competing with the other schools in their area

 

Q. 11 which one of the following is not related to other options?

A. organizing question-answer sessions

B. taking feedback from students on a topic

C. conducting quiz

D. modeling the skills of self assessment

 

Q. 12 which one of the following questions is correctly matched with its specified domain?

A. could you group your : evaluating students on the basis of their achievement in mathematics?

B. what was the turning : creating point in the cricket match telecast last night?

C. write down a new: application recipe to cook chicken by using herbs

D. determine which of the : analyzing given measures would most likely lead to achieve best results

 

Q. 13 which of the following is the most effective way to convey students from disadvantaged sections that you expect them to participate and succeed?

A. articulate your confidence in their ability to succeed

B. develop your own interest in the topics to be taught

C. compare them with other children as frequently as possible to make them realize their goal

D. emphasize the point that you have high expectation of them

 

Q. 14 following are the examples developmental disorder, except

A. autism

B. cerebral palsy

C. post-traumatic stress

D. attention deficit hyperactivity disorder

 

Q. 15 multiple pedagogical techniques, assorted learning material, multiple assessment techniques and varying the complexity and nature of the content are associated with which of the following ?

A. universal design for learning

B. remedial teaching

C. differentiated instruction

D. reciprocal teaching

 

Q. 16 which of the following is true about gifted learners?

A. they make everyone else smarter and are essential for collaborative learning

B. they always lead others and assume extra responsibility in the classroom

C. they may achieve lower grades due to their heightened sensitivity

D. their importance is primarily due to their brain power

 

Q. 17 inclusion in schools primarily focuses on

A. making subtle provisions for special category children

B. fulfilling the needs of children with disabilities

C. meeting the need of the disabled child ate the expense of entire class

D. including the educational needs of illiterate parents in schools

 

Q. 18 the cause of learned helplessness in children is their

A. acquired behaviour that they will not succeed

B. callous attitude towards classroom activities

C. non-compliance with expectations of their parents

D. moral decision for not taking up studies seriously

 

Q. 19 If a student is consistently getting lower grades in schools, her parents can be advised to help her by

A. working in close association with teachers

B. with holding mobile phones, movies, comics and extra time for play

C. narrating her the hardships of life for those who do not possess proper education

D. forcing her to work harder at home

 

Q. 20 which of the following does not deter problem solving?

A. insight

B. mental sets

C. entrenchment

D. fixation

 

Q. 21 A teacher is connecting a text to the previously learnt text and showing children how to summarize it. she is

A. helping children to develop their own strategy to comprehend it

B. insinuating that there is no need to go through the entire text

C. reinforcing the importance of text from the assessment point of view.

D. encouraging children to mug it up effectively as possible

 

Q. 22 what kind of errors is common between a learner who is learning his mother tongue and the learner who learns the same language as a second language?

A. overgeneralization

B. simplification

C. developmental

D. hypercorrection

 

Q. 23 the stress affects performance in examinations . this fact reflects which of the following relationships?

A. cognition-emotion

B. stress-omission

C. performance-anxiety

D. cognition-competition

 

Q. 24 A teacher is trying to counsel a child who is not performing well after an accident. which of the following is most appropriate about counselling in schools?

A. it is about the palliative measures for making people comfortable

B. It builds self-confidence of people by letting them explore their own thoughts

C. it is about giving the best possible advice to students about their future career options

D. it can be done only by the professional experts

 

Q. 25 which of the following would encourage at least a student who wants to become a highly creative theatre artist?

A. try to win the state level competition that will ensure you scholarship

B. develop empathetic, amicable and supportive relationships with your peer theatre artists

C. devote your time to those theatrical skills that you find most enjoyable

D. read about the performances of the worlds best theatre artists and try to learn

 

Q. 26 which of the following theorists would be of the opinion be that students study hard for their personal growth and development?

A. bandura

B. maslow

C. skinner

D. piaget

 

Q. 27 which of the following factors supports leaning in a classroom?

A. increasing the number of tests to motivate children to learn

B. supporting the autonomy of children by the teachers

C. sticking to one particular method of instruction to maintain uniforming

D. increasing the time interval of periods from 40 minutes to 50 minutes

 

Q. 28 mature students

A. believe that emotion has no place in their studies

B. resolve easily all their conflicts with their intellect

C. sometimes need emotional support in their studies

D. do not get upset by studies in difficult situations

 

Q. 29 A child coming to pre-schools for the first time cries profusely. After two years when the same child goes to the primary school for the first time, he does not express his tension by crying rather his shoulder and neck muscles become tense. This change in his behaviour can be can be explained on the basis of which of the following principles? 

A. development proceeds in a sequential manner

B. development is gradual

C. development is different in different people

D. development is characterized by differentiation and integration

 

Q. 30 which of the following statements is true?

A. genetic makeup impacts responsiveness of an individual to qualities of the environment

B. adoptive children possess same IQs as their adoptive siblings

C. experience does not influence brain development

D. intelligence remains unaffected by the schooling

 

Q. 31 the perimeter of a square is 20 cm. A rectangle has the same width as the square. The length of the rectangle is double its width. The area, in square cm, of the rectangle is

A. 30

B. 50

C. 100

D. 25

 

Q. 32 The internal length, breadth and height of a rectangular box A are 20 cm, 18 cm and 15 cm respectively and that of box B are 18 cm, 12 cm and 5 cm respectively. The volume of the box A is how many times that of box B?

A. 4

B. 5

C. 6

D. 3

 

Q. 33 Which of the following is not correct?

A. area of a square of side 10 cm, area of a rectangle of length 10 cm and breadth 0.1 m

B. 1 meter 5 centimeter = 1.5 meter

C. 0.40 is same as 0.4

D. rupee 3/4 = rupee 0.75

 

Q. 34 one orange costs five and half rupees and one kg apple costs 80 rupees then the total cost of one and half dozen of oranges and one and three-fourth kg of apples is

A. rs. 219

B. rs. 229

C. rs. 239

D. rs. 209

 

Q. 35 NCF, 2005 states that mathematics teaching should be ambitious, coherent and important. Here, ‘ambitious’ refers to achievement of

A. narrow goals of mathematics

B. linking of mathematics with other subjects

C. application of mathematics

D. higher goals of mathematics

 

Q. 36 In class II, concept of even number and odd number was introduced by activity of pairing the given number with concrete objects. then teacher asked the students to check 

(i) whether the total number of crayons in their colour boxes is even in number or odd in number,

(ii) whether the number of pages in their maths notebook is odd or even. This task of finding even or odd number of crayons/pages is

A. assessment of learning

B. assessment as learning

C. assessment at the end of learning

D. assessment of learning

 

Q. 37 After explaining the operation of subtraction in class II , teacher drew the following diagram on the board and asked the students to fill in the circles:

the purpose of the exercise is

A. strengthening skill of addition and subtraction

B. summative assessment

C. arranging fun activity for the students

D. mathematization of mind

 

Q. 38 How many rectangles are there in the following figures?

The above question is testing

A. knowledge of learner

B. understanding of learner

C. creativity of learner

D. memory of learner

 

Q. 39 Which of the following activities is meant to enhance problem solving abilities of students of class III?

A. A crossword puzzle containing clues for all key terms learnt like even numbers, odd numbers, composite number, prime number, etc

B. A group project: how the students of primary wing shall be distributed equally in four (house system of school)so that every house has talented students from sports, arts, cultural and academic activities

C. conduct an interclass quiz in the class on ‘numbers and operations’

D. A worksheet containing problems on four basic operations like find 25 x 34, 451÷11 etc.

 

Q. 40 The recommended key concept in the unit of ‘multiplication’ in class III is

A. multiplication of 2 digit number with 2 digit number

B. properties of multiplication–order property and group property

C. word problem based on multiplication

D. multiplication of 3 digit numbers by 10

 

Q. 41 from the unit of ‘fraction’ teacher asked the students to list any five fractions.

this question refers to

A. higher level of thinking

B. analytical thinking

C. spatial thinking

D. lower level of thinking

 

Q. 42 In an exercise, the question was-measure the lengths of the segment P—–Q and R—–S the child answered

length of AB = 5 cm

length of AB = 3 cm

this refers to

A. conceptual error

B. procedural error

C. error due to habit of naming line segment as AB

D. reading error

 

Q. 43 A teacher distributed newspaper in class V and asked the students to read the cricket scores of indian team in latest match. then she asked them to draw bar graph of the scores. the teacher is trying to

A. help the students to make connections between mathematical concepts and real life

B. teach them through project approach

C. make the class joyful and communicating

D. enhance the reasoning power of the students

 

Q. 44 The following word problem was given to a child of class IV:

Mumbai has 336 bus stops. Delhi has 127 more bus stops than Mumbai. How many bus stops are there in all, in Delhi. The child’s response to the above problem is recorded as– total bus stops in Delhi are 336+127 = 463

the teacher shall report about his performance as


A. the child can find the answer correctly but his expression is poor

B. the child is good at visualizing and analyzing , and his problem solving ability is appreciable

C. the child has not adopted the right procedure, he needs to practise more

D. the child is not able to write all steps required properly

 

Q. 45 A student was asked to read the following numbers:

306, 408, 4008, 4010. He read as follows:

thirty six, forty eight, four hundred eight, forty ten

the reason for error in reading is that

A. the student does not like mathematics class and finds the class boring

B. the student has understood the concept of place value but does not know how to use it C. the student is not fit for study of maths

D. the student is not able to understand the concept of place value and feels comfortable using two-digit numbers only

 

Q. 46 The parameter(s) of assessment for ‘time’ for class IV shall be

A. reading time on analogue clock only

B. reading time on digital and analogue clock, concept of half past, quarter past, quarter to, a.m , p.m , relation between minutes and seconds.

C. reading time on digital clock , concept of a.m and p.m only

D. reading time on digital clock only

 

Q. 47 Tall shape of mathematics mentioned in NCF, 2005 refers to

A. solving challenging problems

B. creating maths game

C. providing hands on experience

D. building of one concept on other

 

Q. 48 consider the following:

5 + 3 = ?

the open minded question corresponding to the above close ended question is

A. find the sum of 5 and 3

B. what shall be added to 5 to get 8 ?

C. give any two numbers whose sum is 8

D. what is the sum of 5 and 3

 

Q. 49 Farhan went to school library and found that 100 books kept in story section are spoiled. 20 books are missing. 219 are kept in shelf and 132 were issued to students. How many story books were there in the library?

teacher can teach the following value through this question

A. helping others

B. sharing books with others

C. taking good care of books

D. sense of cooperation

 

Q. 50 The product of the place values of 5 and 6 in 70560 is

A. 300

B. 3000

C. 30000

D. 30

 

Q. 51 when 3010301 is divided by 43, the quotient is

A. 70707

B. 70007

C. 7077

D. 7007

 

Q. 52 what should be subtracted from the product 3001 x 101 to get 300311?

A. 2790

B. 2090

C. 2970

D. 270

 

Q. 53 (smallest common multiple of 36 and 60)+(largest common factor of 18 and 45) is equal to 

A. 20

B. 30

C. 40

D. 10

 

Q. 54 (10 tens + 11 hundreds + 12 ones) equals

A. 1213

B. 111012

C. 101112

D. 1212

 

Q. 55 the number of factors of 42 is

A. 7

B. 8

C. 9

D. 6

 

Q. 56 when 3488 is divided by 12 and 2478 is divided by 11, the difference between the remainders in both cases is

A. 5

B. 6

C. 7

D. 3

 

Q. 57 A train leaves P at 8:18 a.m and reaches station Q at 10:28 p.m. on the same day. The time taken by the train to reach Q is

A. 14 hours 10 minutes

B. 14 hours 46 minutes

C. 18 hours 46 minutes

D. 13 hours 10 minutes

 

Q. 58  14 L 280 ml of orange juice and 18 l 830 ml of carrot juice were mixed together. This mixture was filled in 15 bottles each containing 1.5 L. How much mixture was left?

A. 11 L 105 mL

B. 9 L 610mL

C. 11 L 60 mL

D. 10 L 610 mL

 

Q. 59 One fourth of a pizza was eaten by Renu. The rest was equally distributed among 12 children. what part of the pizza did each of three children get?

A. 1/16

B. 1/32

C. 3/16

D. 1/8

 

Q. 60 The number of seconds in 6 hours equals the number of minutes in

A. 4 days

B. 10 days

C. 15 days

D. 2 days

 

Q. 61 select the true statements from the following:

I. elephants love to play with muddy water as it keeps their skin cool

II. most elephants like to take rest and sleep nearly 10 hours a day.

III. A three month old elephant weighs about 200 kg

IV. most adult elephants eat about 100 kg of leaves/twigs in one day.

 

A. II and IV only

B. I and IV only

C. I, III and IV only

D. I and II only

 

Q. 62 select the correct statements about madhubani paintings:

I. to make these paintings, indigo, turmeric, colours from flowers and trees, etc..are used

II. The paintings show human beings, animals, trees, flowers, birds, fish etc

III. it is a very old from of folk art and is named after the place madhubani

IV. madhubani is a well-known district of Rajasthan

A. I, III and IV

B. I, II and IV

C. II, III and IV

D. I, II and III

 

Q. 63 A bird makes its nest high up on a tree. This bird could be

A. sun bird

B. indian robin

C. crow

D. dove

 

Q. 64 Pochampally is a village which is famous for the special cloth which is also called pochampally. This village is a part of

A. kerala

B. karnataka

C. andhra pradesh

D. tamil nadu

 

Q. 65 select the correct statements about desert oak.

A. it is a tree found in Australia.

B. this is a special kind of tree which has its roots growing from its branches.

C. The roots of this tree go deep till they reach water

D. This tree stores water in its truck . Local people use thin pipe to drink this water

 

A. A, B and D

B. A, C and D

C. B, C and D

D. A, B and C

 

Q. 66 Select the correct statements about Abu dhabi:

A. it is located in desert area

B. water is costlier than petrol in Abu dhabi

C. Arabic is the local language here.

D. Money used in Abu Dhabi is called Dinar.

 

A. A, B and D

B. A,C and D

C. B,C and D

D. A, B and C

 

Q. 67 There are animals that awake night. These animals can see things only in

A. violet and blue colours

B. green and yellow colours

C. black and white colours

D. red and orange colours

 

 

Q. 68 Here in fig. I, a 24-hour clock is shown and in fig. II and fig III sleeping times of two animals X and Y are shown by shaded portions. select the correct statement about the sleeping times X and Y

A. X sleeps for 20 hours while Y sleeps for 18 hours

B. X sleeps for 18 hours while Y sleeps for 4 hours

C. X sleeps for 4 hours while Y sleeps for 18 hours

D. X sleeps for 4 hours while Y sleeps for 20 hours

 

Q. 69 Which is the dead sea?

A. A sea whose water is poisonous

B. a sea which is saltiest of all oceans and seas

C. a sea in which high and low tides are very frequent

D. a sea in which it is dangerous to sail ships

 

Q. 70 anjali desires to emphasize on cultural/regional diversity in food habits, while teaching the topics on ‘food’ to class III students. which one of the following learning experiences given to students bring about the desired result?

A. use a chart showing foods of different cultures/ regions and explain

B. ask students from different cultures/ regions to bring food from home, display it and share the information with classmates

C. explain the information given in the text book about different foods in greater detail

D. request an external professional cooking agency to display food of different regions/ cultures

 

Q. 71 Preeti wants to lay greater emphasis on pollution while teaching environmental concerns to class V students. Which one of the following activities is likely to be most effective in achieving the desired objective?

A. asking students to prepare charts on different kinds of pollution

B. taking students on a field visit to a polluted river

C. asking students to take up group projects related to different kinds of pollution

D. inviting experts to talk on air, water and noise pollution

 

Q. 72 neha uses the following assessment techniques in the subject of EVS for class V:

I. hands on activity assessment

II. home assignment assessment

III. project work assessment

IV. oral testing

Which one of the following pairs of techniques is likely to be more objective assessment?

A. II and III

B. I and IV

C. II and IV

D. I and II

 

Q. 73 A good home assignment in EVS should primarily focus on

A. revision and reinforcement

B. mastery learning

C. challenge and excitement for extended learning

D. better utilization of time

 

Q. 74 Shalini has planned a field trip for class IV students to the science centre. which one of the following general instructions given to the students is irrelevant for the trip?

A. ask questions for your doubts on displays

B. carry your full school bag for the day

C. do not go anywhere without informing me

D. take a notepad and pen with you

 

Q. 75 Good EVS curriculum at primary stage should

A. include more practise questions in end exercises

B. provide opportunities to explore surroundings

C. focus more on detailed explanation of concepts

D. emphasize more on exact definition of terms

 

Q. 76 poems and stories have been included in EVS text books for primary stage in order to

A. develop literary skills in students

B. provide fun and enjoyment in learning of the subject

C. enhance understanding of fundamental concepts

D. have a change in routine and monotony of presentation of content

 

Q. 77 four perspective teachers were asked to make a presentation on the theme travel to class V students each of them mainly focussed on one of the following strategies 

A. using charts to show different modes of travel along with explanation

B. using textbook content to explain different modes of travel

C. asking students to collect pictures of different modes of travel and prepare a scrapbook

D. asking students to narrate their own travel experiences using various modes of travel 

 

Q. 78 think and discuss section in one chapter of EVS textbook in class V includes the following statement:

what would happen if you don’t get petrol or diesel for a week in your village or town the statement primarily aims at

A. sensitizing students on using oil judiciously

B. assessing students on sources of petrol and diesel

C. promoting imaginative and thinking skills to understand real life concerns

D. creating awareness about scarcity of petrol and diesel

 

Q. 79 The technique of classrooms questioning in teaching of EVS can be used best for

A. drawing attention of students

B. arousing curiosity in the learners

C. promoting practical skills

D. maintaining discipline in the class

 

Q. 80 one of the major objectives of teaching of EVS at primary stage is to

A. develop in depth understanding of basic concepts of the subject

B. prepare students for studies at the next stage

C. help the learners link classroom learning to life outside the school

D. acquire skills to carry out hands on activities independently

 

Q. 81 which one of the following is not a suitable activity at primary stage to sensitize students to the concept of conservation of trees

A. organizing a slogan writing competition on trees

B. encouraging every student to adopt a tree and look after it

C. showing children storage of logs of wood

D. organizing a poster making competition on trees.

 

Q. 82 while teaching the topic on water in her EVS classroom, Anjali organizes role-play on different sources of water and individual actions to conserve water. The activity to primarily aimed at

A. improving social skills of students

B. breaking monotony in the process of learning

C. ensuring active participation of students in the process of learning

D. enhancing students knowledge on sources of water

 

Q. 83 EVS curriculum at primary stage has been developed to include pure science as well as social science concepts. This has been done primarily to

A. enable a learner look at environment in a holistic manner

B. reduce the number of subjects that are studied

C. reduce the load of school bag

D. reduce requirement of subject teachers

 

Q. 84 which one of the following will be more effective learning experience to emphasize more on social inequalities in an EVS class?

A. showing video films on related issue

B. organizing special lectures on related issue

C. conducting a quiz contest on the issue

D. asking the students undertake group projects

 

Q. 85 while discussing liking and disliking a student says , I and my mother both love to eat snakes. whenever we feel like eating snakes, we go to a nearby hotel and eat ling-hu-fen. this student must belong to

A. assam

B. hong kong

C. odisha

D. arunachal pradesh

 

Q. 86 study the following statements about braille script.

I. braille is written on a thick paper by making dots(raised points)

II. this script is based on eight points

III. rows of dots are made with a pointed tool

IV. it is read by running the fingers on the raised dots.

the correct statements are

A. I, III and IV

B. I, II and IV

C. II, III, and IV

D. I, II and III

 

Q. 87 Select the correct statements from the following:

A. animals that have outside ears and hair on their body gave birth to the young ones.

B. animals that do not have outside ears and hair on their body lay eggs.

C. animals that do not have outside ears and hair on their body give birth to the young ones

D. animals that have outside ears and hair on their body lay eggs

 

A. A and C only

B. B and D only

C. B and C only

D. A and B only

 

Q. 88 Select the correct statements about beekeeping:

I. january to march is the best time to start beekeeping.

II. honey bees are attracted to the lichi flowers

III. boxes are needed for keeping bees and storing honey produced by them

IV. sugar is purchased to make syrup for honeybees.

A. II and III only

B. I and IV only

C. II, III and IV

D. I and II only

 

Q. 89 A student noted down the following information in her diary about her journey from ahmedabad to nagercoil:

from the above information the following conclusions were drawn:

A. the distance between ahmedabad and nagarcoil is 2348 Km.

B. The train covers this distance in 40 hours 55 minutes.

C. the train covers this distance in 40 hours 55 minutes

D. the average speed of the train is about 80 Km per hour.

The correct conclusions are

A. A, C and D

B. A, B and D

C. A and B only

D. A and C only

 

Q. 90 which of the following information may be obtained from a reserved rail ticket/

A. bate and time of booking, date and time of the start of journey

B. coach number, berth number and fare

C. train number with name, boarding station and last station.

D. date and time of arrival at the destination.

E. date and time of arrival at the destination

 

A. A, B and D

B. A, B, C and D

C. A, B , D and E

D. A,C and E only

 

Questions: 91 – 99

He has reservations on the treatment of dance in the Indian films, but given a chance to work on his own terms, legendary kathak dancer pandit birju maharaj would like to work more n bollywood. To make it more dramatic, the dancers are asked to perform in an exaggerated manner. That makes any kind of dance impure, especially classical dance. I’d like to work more in Hindi films, provided my dance is not tampered with. the kathak maestro tells us that over the years he’s been highly impressed with how some female actors have showcased classical dance on screen. On being asked on how he sees the passion for dance among youngsters in the country, birju maharaj says, I see that the young generation is divided in their response to classical dance. But in all my interactions with the younger lot, I have been impressed. These children have such amazing presence of mind, listening and learning while I talk and teach them. It is often said that classical dance does not receive due credit, but the man who is an authority on the subject thinks Delhi receives the art well. I fell that classical dance might not be on a rise, in popularity , but I have always been overwhelmed by the response that I have received in Delhi. My performances have always been applauded by packed houses in the capital, he opines.

 

Q. 91 The information presented here about birju maharaj can be found in a/an

A. newspaper article

B. diary

C. encyclopedia

D. autobiography

 

Q. 92 The observation that dance is adulterated means that the dance form is

A. not practiced according to tradition

B. found in adult entertainment

C. performed only in films

D. suitable to be performed by adults

 

Q. 93 here to perform is an exaggerated manner suggests that performers

A. are not professionally trained

B. deliberately distort the dance form

C. only dance for a selected audience

D. cannot dance

 

Q. 94 A packed house during his performance suggests that it was

A. jammed in tightly

B. filled into

C. exceeding allotted time

D. well attended

 

Q. 95 The younger dancers have presence of mind means that they

A. are open to learning the pure form of the dance

B. prefer traditional styles of dancing

C. can combine to perform in the traditional and modern styles

D. are calm while they prepare to perform

 

Q. 96 Birju maharaj assessment of his popularity lies in

A. the large numbers of practitioners and admirers of his style in Delhi

B. the influence of traditional styles in modern dance

C. his mirating to mumbai on popular demand by producers

D. the number of dances he has choreographed in films

 

Q. 97 A word that can replace the phrase tampered with in the passage is

A. falsified

B. misrepresented

C. disturbed

D. misused

 

Q. 98 An antonym for the word showcased is

A. advertised

B. published

C. abridged

D. withheld

 

Q. 99 A synonym for the word inspired from the text is

A. adulterated

B. impressed

C. received

D. divided

 

Q. 100 to evaluate the reading comprehension at class III level, students may be asked to

A. retell a story in their own words or to summarize the main idea or the moral of the story

B. take a short dictation of what they have read

C. write a short diary based on one of the characters

D. use a set of identified vocabulary on their own

 

Q. 101 An exercise where words are left out of a shorter passage and the pupil must fill in the blanks with suitable words based on her reading assesses her ability to

A. comprehend

B. use new words

C. summarize

D. spell words

 

Q. 102 creating or retrieving what the student wants to say then generating a suitable text to say it, are stages of the______process

A. listening

B. writing

C. speaking

D. creative

 

Q. 103 use of grammar punctuation and spelling pertains to

A. text production while writing

B. formal speech

C. listening to a lecture

D. informal conversation

 

Q. 104 Read the following exchange

speaker 1 : have you been to indore?

speaker 2 : who?

speaker 1 : to indore in madhya pradesh

speaker 2 : umm.. I am not sure

during the assessment of students speaking skills marks would be deducted during this exchange for

 

A. none of them

B. the second speaker

C. both of them

D. the first speaker

 

Q. 105 A student recommends the reading of the latest best seller saying that it is very interesting. you listen trying to make out whether the students observation is a sincere or not. This type of listening can be described as

A. critical

B. comprehension

C. sympathetic

D. active

 

Q. 106 children deserve most of the credit for language that they acquire. This observation implies that in modern classrooms

A. students pursue their own lines enquiry

B. students need not attend L2 classes

C. students may choose L2 on their own

D. the teacher establishes the task and supports or facilitates learning

 

Q. 107 types of text media are

A. audio discs and tapes

B. illustrations and diagrams

C. motion pictures and documentaries

D. digital e-books, e-journals

 

Q. 108 which of the following resources will help to break down communication barriers and enable children to study and learn in both L1 and L2?

A. multilingual

B. multimedia

C. more textual

D. commutative

 

Q. 109 students learning a language often lack confidence when speaking due to the languages unique pronunciation rules. One way to overcome this problem

A. children reading aloud in class

B. using game like activities which require verbal interactions in the classroom

C. conducting special speech therapy with a counsellor

D. correcting errors whenever they happen

 

Q. 110 The contemporary target language classroom is a confluence of varied languages and language abilities. Teachers should restructure their practices by exposing students to 

A. summaries and simplified versions of the learning materials, e.g stories, grammar, notes etc

B. worksheets with a variety of tasks which cover the syllabus, and students give their responses in class under teachers guidance

C. appropriate challenges based on the syllabus in a secure environment opportunities for all students to explore ideas and gain mastery

D. adequate self explanatory notes, either prepared by teacher or from material writers 

 

Q. 111 for students to gain language skills from textbooks, the textbook learning should 

A. correlate with assessment and achievement

B. lead to using the textbook sparingly

C. expose them to more literary reading

D. become more cost-effective compared to technologically supported courses

 

Q. 112 while teaching hearing impaired students in an inclusive class, it is necessary for teachers to

A. make sure that they are including signs and non verbal signals to strengthen any communication

B. conduct regular a special class for such students

C. be in constant touch with the parents of such students

D. use cue cards to signal the teaching content

 

Q. 113 while translating a subject and using the translation in the mainstream curriculum, the benefit is

A. promoting national identity

B. enriching linguistic capability and appreciation

C. enabling teachers who are not complement in the mainstream language to take classes

D. standardizing cultural identify

 

Q. 114 some criteria for the selection of language items should involve

A. enough worksheets for practise

B. everyday vocabulary and sentences

C. their learnability coverage and teachability

D. a focus on language rules

 

Questions: 115 – 120

sprinkle, squish between my toes,

the smell of ocean to my nose.

I can feel each grain of sand,

It falls from air into my hand.

the shells i find along the shore,

picked up by birds that fly and soar.

they sparkle like the oceans waves,

and carry sand from all the lakes.

I walk

That’s where my feet leave prints to be.

I walk all the way to the end of the land,

the land that holds this beautiful sand

 

Q. 115 The poem’s central theme is

A. a factual description of nature

B. sharing experience with nature

C. a recollection of a visit

D. an introspection by the writer

 

Q. 116 Here to the end of the land refers to the

A. sealine

B. land

C. sky

D. horizon

 

Q. 117 here, that’s where my feet leave prints to be means that the writer

A. knows that everything is temporary

B. relives past visits

C. expects to forget the experience

D. hopes to remember his visit

 

Q. 118 The phrase in the poem that conveys the same meaning as along the tip of the sea is 

A. each grain of sand

B. end of the land

C. air into my hand

D. like the oceans waves

 

Q. 119 The poetic device used in the line. They sparkle like the oceans waves is a/an

A. hyperbola

B. exaggeration

C. simile

D. allegory

 

Q. 120 A word that can replace squish is

A. crush

B. hold

C. scrunch

D. trample

 

Questions: 121 – 129

“Get well soon !” shanta said, handling partha a yellow balloon. she was his third visitor. That’s because she was his third visitor. that’s because she was his class teacher’s daughter, and her mother made her visit him. The other two, Rahul and syed , weren’t really his friends , although they often granted up with him against other kinds to take away their lunch pocket money. Partha knew he wouldn’t have long to live. He could feel it, deep inside. seeing his aunty crying after talking with the doctor confirmed it. his time had come. He didn’t tell his visitors, though. They would either pity him, or be happy to get rid of him. once shantha left, he ripped a page off from his notebook and wrote– “Dear god, I know I messed up and nobody likes me. Please give me a second chance. I show you what a good friend can be.” He drew a map showing the way from the school to the hospital, walked shakily to the window and let the balloon fly away, carrying his message towards god. The balloon was heading straight to a telephone pole, but a gentle breeze blew it away just in time. It crossed the park and disappeared out of view. The next day, a boy he had never met before came to visit him. “I find balloon “,he said. “You are lonely?”

he just nodded, too startled to talk. ”I lonely too. My family come from Afghanistan and I no speak English good.” He smiled. “I bring gift to you.” He handed him a small bag of fruits. “I pray for friend, and god give me friend.” Normally, he would have made fun of his broken english and his long, baggy brown kurta, but he knew better. He smiled and offered him the first orange. 

 

Q. 121 one student the reader understands did not really want to meet partha that is

A. shantha

B. syed

C. rahul

D. the afghan boy

 

Q. 122 Partha felt lonely because _____visited him when he was at the hospital.

A. his class teacher

B. only syed and rahul from his class

C. he had no friends at school, so no one

D. shantha

 

Q. 123 A synonym for the word Startled in the passage

A. surprised

B. stunned

C. avoided

D. composed

 

Q. 124 An antonym for the word shakily in the passage

A. unsure

B. unsteady

C. firmly

D. rickety

 

Q. 125 Parthas feeling of loneliness soon turned to

A. irritation

B. sympathy

C. self pity

D. anger

 

Q. 126 partha would not go back to school to meet his schoolmates and teachers because 

A. his parents wanted to change his school

B. he did not have long to live

C. he hated his school as he had no friends

D. his doctors did not let him

 

Q. 127 The message in the passage is

A. loneliness is inevitable

B. about faith in god

C. all actions have consequences

D. friendship is rare

 

Q. 128 The change in parthas attitude is evident when he

A. accepted the fruits from a stranger though he disliked oranges

B. refrained from poking fun at the afghan boy and shared the fruits

C. was unmoved even when his aunt was crying

D. sent a letter to god written on a balloon

 

Q. 129 A phrase that can replace the words ganged up is

A. joined in opposition

B. formed a group

C. supported together

D. became friends

 

Q. 130 while evaluating students response for reading comprehension , marks may be deduced for errors

A. grammatical

B. syntactical

C. content

D. spelling

 

Q. 131 A text that requires students to scan in order to understand and analyze the writer’s message and purpose could be

A. an encyclopedia extract

B. a newspaper headline

C. a set of instructions to assemble a device

D. a poem

 

Q. 132 while drafting a notice, students may be instructed to use____language

A. direct

B. intrigue

C. descriptive

D. elaborate

 

Q. 133 For evaluating a poster designed as a part of a competition, which of the following criteria would be the most appropriate for the judges?

A. relevant content, style and word limit

B. use of quotations style and visual appeal

C. creativity relevant content and visual appeal

D. creativity fonts and visual appeal

 

Q. 134 for evaluating poster designs as a part of a competition, which of the following criteria would be the most appropriate for the judges ?

A. relevant content, style and word limit

B. use of quotations, style and visual appeal

C. creativity, relevant content and visual appeal

D. creativity, fonts and visual appeal

 

Q. 135 In order to drive home the point that listening skills matter, students should be 

A. exposed to popular english films and reviewing them

B. taken to the languagelab once a week for practise aloud by well-known voice artists 

C. allowed to listen to CD’s of poetry read aloud by well known voice artists

D. reminded that listening skill practices relate to real life

 

Q. 136 learning a new language after puberty leads to ______of a foreign language

A. difficulty in acquisition

B. normal acquisition

C. greater mastery

D. loss of mastery

 

Q. 137 An activity that requires a class to design and present a powerpoint on the importance of water conservation in a target language is a ____activity.

A. multidisciplinary

B. science project

C. group

D. language practise


Q. 138 To enable students to distinguish between academic and spoken forms of a target language in a bilingual class, they should be encouraged to

A. speak in the target language

B. read more books written in the target language

C. write more in the target language

D. watch more related bilingual films

 

Q. 139 providing learning support to pupils who lag far behind their counterparts in school performances includes

A. giving more activities for language practise

B. providing extra notes and coaching

C. allowing them to complete assignments without time limits

D. initially adapting school curricula and teaching strategies

 

Q. 140 enriching the curriculum for learners who are gifted and talented

A. give them leadership roles in class activities

B. increase complexity of curriculum for them to experience a wider variety of language and opportunities for creativity

C. promote them to a higher class so that they are exposed to a more difficult syllabus

D. introduce a foreign language

 

Q. 141 identify the false assumption:

language course books prescribed for students should provide’

A. detailed lesson plans for teachers

B. suggested sequence of teaching procedures

C. balanced presentation of information

D. organized units of learning experience

 

Q. 142 students who do not have the opportunities to use the target language outside the classroom, demonstrate much lower levels of language competency. This can be overcome by 

A. conducting tests periodically to motivate them to learn

B. giving them a set of commonly used sentences and vocabulary which they are expected to use

C. setting separate tasks which are easier, with more time to complete them

D. engaging them in specific language focused tasks which are in =directly monitored by their group leaders

 

Q. 143 A foreign/ non mother tongue language teacher often faces the problem of a class full of reluctant, unmotivated learners. This can be helped by

A. using methods and strategies to motivate and make learning more challenging in the class

B. taking the help of an academic counsellor who will address the class

C. identifying the students who are unmotivated and taking a special class for them

D. encouraging learners to take their own time to complete assignments

 

Q. 144 Language learners learn to do by doing. which activity supports this observation?

A. opportunities to practise as it helps with habit formation

B. encourage the use of their mother tongue to promote better understanding of the meaning of a prescribed text

C. go from concrete to abstract texts

D. the teacher models the writing and speaking styles which learners copy

 

Questions: 145 – 150

our consumption of palm oil is rocketing commitments from various governments to increase the amount of biofuels being sold are pushing this rise in demand, because they re sold seen as an attractive quick fix to reduce greenhouse gas emissions. India wanted 20% of its diesel to be biodiesel by 2012. The irony is that these attempts to reduce the impact of climate change could actually make things worse–clearing forests and draining and burning peatlands to grow palm oil which releases more carbon emissions than burning fossil fuels. but this phenomenal growth of palm oil industry spells disaster for local communities, biodiversity, and climate change as palm plantations encroach further and further into forested areas where the emissions of greenhouse gases is largely due to deforestation. for example, much of the current and predicted oil palm expansion is taking place on forested peatlands. peat locks up huge amounts of carbon. so clearing peatlands by draining and burning releases huge greenhouse gases. The roundtable on sustainable palm oil (RSPO) has established clear ethical and ecological standards for producing palm oil. but since then, forest destruction has continued unabated.

 

Q. 145 The passage is about the impact of _____on environment

A. destruction of peatlands

B. palm oil industries

C. drilling for diesel fuel

D. loss of communities

 

Q. 146 Peatlands are natural

A. means to suppress carbon emissions

B. causes of environmental destruction

C. sources of biofuels when burnt

D. sources of diesel

 

Q. 147 The phrase in the passage means speedy remedy is

A. phenomenal growth

B. quick fix

C. current and predicted

D. draining and burning

 

Q. 148 The synonym of the word irony is

A. respect

B. reality

C. praise

D. paradox

 

Q. 149 The RSPO was converted to

A. control destructive practises in palm oil production

B. control the burning of peatlands

C. rehabilitate local communities

D. force the closure of palm oil industries

 

Q. 150 The passage suggests that RSPO’s efforts to carry out its responsibility has been

A. mostly successful

B. mostly a failure

C. no information in the passage

D. partly successful

 

Answer Sheet 
Question 1 2 3 4 5 6 7 8 9 10
Answer B B A B B D C A D B
Question 11 12 13 14 15 16 17 18 19 20
Answer D D A C C A A A A A
Question 21 22 23 24 25 26 27 28 29 30
Answer A C A B A B B C D A
Question 31 32 33 34 35 36 37 38 39 40
Answer B B B C D A A B B B
Question 41 42 43 44 45 46 47 48 49 50
Answer D C A B D B D C C C
Question 51 52 53 54 55 56 57 58 59 60
Answer B A A D B A A D A C
Question 61 62 63 64 65 66 67 68 69 70
Answer C D B C B D C C B B
Question 71 72 73 74 75 76 77 78 79 80
Answer C B C B B B D C B C
Question 81 82 83 84 85 86 87 88 89 90
Answer C C A D B A D C C A
Question 91 92 93 94 95 96 97 98 99 100
Answer A A B D A A B D B A
Question  101 102 103 104 105 106 107 108 109 110
Answer A B A B A A D A B C
Question  111 112 113 114 115 116 117 118 119 120
Answer A A B C B A D B C D
Question 121 122 123 124 125 126 127 128 129 130
Answer A C B C C B C B A C
Question 131 132 133 134 135 136 137 138 139 140
Answer D A C A D A A B D B
Question 141 142 143 144 145 146 147 148 149 150
Answer A D A A B A B D A B

CTET July 2013 Paper-II Previous Year Paper

CTET July 2013 Paper-II

Q. 1 Which one of the following is a critique of theory of multiple intelligences ?

A. Multiple intelligence are only the ‘talents’ present in intelligence as a whole.

B. Multiple intelligence provides students to discover their propensities.

C. It over emphasises practical intelligence.

D. It cannot be supported by empirical evidence at all.

 

Q. 2 Which one of the following pair is least likely to be a correct match ?

A. Children enter in – Chomsky the world with certain knowledge about language

B. Language and ·-. Vygotsky thought are initially two different activities

C. Language IS· — Piaget contingent on thought

D. Language IS a – B.F. Skinner stimuli in environment

 

Q. 3 Features assigned due to social roles and not due to biological endowment are called

A. Gender role attitudes

B. Gender role strain

C. Gender-role stereotype

D. Gender role diagnosticity

 

Q. 4 Which of the following will be most appropriate to maximise learning ?

A. Teacher should identify her cognitive style as well as of her students’ cognitive style.

B. Individual difference in students should be smoothened by pairing similar students.

C. Teacher should focus on only one learning style to bring optimum result.

D. Students of similar cultural background should be kept in the same class to avoid difference in opinion.

 

Q. 5 All of the following promote assessment as learning except

A. telling students to take internal feedback.

B. generating a safe environment for students to take chances.

C. tell students to reflect on the topic taught.

D. testing students as frequently as possible.

 

Q. 6 When a cook tastes a, food during cooking it may be akin to

A. Assessment of learning

B. Assessment for learning

C. Assessment as learning

D. Assessment and learning

 

Q. 7 Differentiated instruction is

A. using a variety of groupings to meet student needs.

B. doing something different for every student in the class.

C. disorderly or undisciplined student activity.

D. using groups that never change

 

Q. 8 In a culturally and linguistically diverse classroom, before deciding whether a student comes under special education category, a teacher should

A. Not involve parents as parents have their own work

B. ‘Evaluate student on her/his mother language to establish disability

C. Use specialised psychologists

D. Segregate the child to neutralise environmental factor

 

Q. 9 Learning disabilities may occur due to all of the following except ·.

A. Teachers way of teaching

B. Prenatal use of alcohol

C. Mental Retardation

D. Meningitis during infancy

 

Q. 10 An inclusive school reflects on all the following questions except :

A. Do we believe that all students can learn

B. Do,we work in teams to plan and deliver learning enabling environment

C. Do we properly segregate special children from normal to provide better care

D. Do we adopt strategies catering for the diverse needs of students

 

Q. 11 Gifted students are

A. Convergent thinkers

B. Divergent thinkers .

C. Extrovert

D. Very hard working

 

Q. 12 The shaded area represent students in a normal distribution who fall

A. At σ=0

B. Between 2σ-3σ

C. After 3σ

D. Between σ-2σ

 

Q. 13 Which one of the following pair would be most appropriate choice to complete the following sentence ? Children faster when they are involved in the activities that seem to be________

A. Forget; useful in a classroom

B. Recall; linked with their classwork only

C. Memorise; culturally neutral

D. Learn; useful in real life

 

Q. 14 CBSE prescribed group activities for students in place of activities for individual students.

The idea behind doing so could be

A. to overcome the negative emotional response to individual competition which may generalise across learning.

B. to make · it easy for teachers to observe groups instead of individual students.

C. to rationalise the: time available with schools most of which do not have enough time for individual activities.

D. to reduce the infrastructural cost of the activity.

 

Q. 15 The conclusion ‘Children can learn violent behaviour depicted in movies’ may be derived on the basis of the work done by which of the following psychologist ?

A. Edward L. Thorndike

B. J.B. Watson

C. Albert Bandura

D. Jean Piaget

 

Q. 16 Students observe fashion shows and try to imitate models. This kind of imitation may be called

A. Primary simulation

B. Secondary simulation

C. Social learning

D. Generalisation

 

Q. 17 If students repeatedly make errors during a lesson, a teacher should

A. make changes in instruction, tasks, timetable or seating arrangements.

B. leave the lesson for the time being and come: back to it after some time.

C. identify the erring students and talk to principal about them.

D. make erring students stand outside the classroom.

 

Q. 18 Following are some techniques to manage anxiety due to an approaching Examination; except

A. familiarising with the pattern of question paper.

B. thinking too much ·about the result.

C. seeking support.

D. emphasising strengths.

 

Q. 19 Bloom’s taxonomy is a hierarchical organisation of ____

A. achievement goals

B. curricular declarations

C. reading skills

D. cognitive objectives

 

Q. 20 A, B and’ C are three students studying ·English. ‘A’ finds it . interesting -and thinks it will be helpful for her in future. ‘B’ studies English as she wants to secure first rank in the class. ‘C’ studies it as she is primarily concerned to secure passing grades. The goals of A, B and C respectively are

A. Mastery, Performance, Performance A voidance

B. Performance, Performance A voidance, Mastery

C. Performance Avoidance, Mastery, Performance

D. Mastery, Performance Avoidance, Performance

 

Q. 21 Even though this was clearly in violation of his safety needs, Captain Vikram Batra died fighting in the Kargil War while ·protecting his country. He might have

A. sought novel experience.

B. achieved self-actualisation.

C. ignored his belongingness needs.

D. wanted to earn a good name to his family.

 

Q. 22 Extinction of a response is more difficult following ·

A. partial reinforcement

B. continuous reinforcement

C. punishment

D. verbal reproach

 

Q. 23 Mastery orientation can be encouraged by

A. focusing on students individual effort.

B. comparing students successes with each other.

C. assigning lot of practice material as home assignments.

D. taking unexpected tests.

 

Q. 24 Which one of the following is correctly matched?

A. 1

B. 2

C. 3

D. 4

 

Q. 25 All the following facts indicate that a child is emotionally and socially fit in a class except

A. develop good relationships with peers

B. concentrate on and persist with challenging tasks

C. manage both anger and joy effectively

D. concentrate persistently on competition with peers

 

Q. 26 Which of the following statements support role of environment in the development of a child ?

A. Some students quickly process information while others in the same class do not.

B. There has been a steady increase in students’ average performance on IQ tests in last few decades.

C. Correlation · between IQs of identical twins raised in different . homes is as high as.0.75.

D. Physically fit children are often found to be morally good.

 

Q. 27 Socialisation includes cultural transmission and

A. discourages rebellion.

B. development of individual, personality.

C. fits children into labels.

D. provides emotional support.

 

Q. 28 A teacher shows two identical glasses filled with an equal amount of juice in them. She empties them in two different glasses one of which is taller and the other one is wider. She asks her class to identify which glass would have more juice in it. Students reply that the taller glass has more juice. Her students have difficulty in· dealing wit 

A. Accommodation

B. Egocentrism

C. Decentring

D. Reversibility

 

Q. 29 Karnail Singh does not pay income tax despite legal procedures and expenses. He thinks that he cannot support a corrupt government which spends millions .of rupees in building unnecessary dams. He is probably in which state of Kohlberg’s stages of . moral development

A. Conventional

B. Post Conventional

C. Pre Conventional

D. Para Conventional

 

Q. 30 Intelligence theory incorporates the mental processes involved in intelligence (i.e. metacomponents) and the varied forms that intelligence can take (i.e. creative intelligence)

A. Spearman’s ‘g’ factor

B. Sternberg’s triarchic theory of intelligence

C. Savant theory of intelligence

D. Thurstone’s primary mental abilities

 

Q. 31 The number of integers less than -3 but greater than -8 is

A. 2

B. 3

C. 4

D. 6

 

Q. 32 The distance between two places is 12 km. A map scale is 1: 25000. The distance between the two places on the map is

A. 24

B. 36

C. 48

D. 60

 

Q. 33 The reciprocal of -(3/8) x -(7/13) is

A. 104/21

B. -104/21

C. 21/104

D. -21/104

 

Q. 34 The number of vertices in a polyhedron which has 30 edges and 12 faces is

A. 12

B. 15

C. 20

D. 24

 

Q. 35 When half of a number is increased by 15, the result is 39. The sum of digits of the original number is

A. 6

B. 7

C. 9

D. 12

 

Q. 36 In ΔPQT, PQ =PT. The points R and S are on QT such that PR = PS. If ∠PTS = 62° and ∠RPS = 34°, then measure of ∠QPR is

A. 11°

B. 13°

C. 15°

D. 17°

 

Q. 37 If for ΔABC and ΔDEF, the correspondence CAB <—> EDF gives a congruence, then which of the followIng is not true ~·

A. AC=DE

B. AB=EF

C. ∠A=∠D

D. ∠B=∠F

 

Q. 38  40% of (100 – 20% of 300) is equal to

A. 16

B. 20

C. 64

D. 140

 

Q. 39 HCF of two numbers is 28 and their LCM is 336. If om~ number is 112, then the other number is

A. 56

B. 70

C. 84

D. 98

 

Q. 40 If 2x/3 = 0.6 and 0.02 y = 1, then the value of x +y-1 is

A. 0.92

B. 1.1

C. 49.1

D. 50.9

 

Q. 41 If Y = (x+2)/(x + 1), y is not equal to 1 and then value of x is

A. (y+2)/(1-y)

B. (y+2)/(y-1)

C. (y-2)/(y+1)

D. (2-y)/(1-y)

 

Q. 42 A square and a circle have equal perimeters. Tine ratio of the area of the square to the area of the circle is

A. 1:1

B. 1:4

C. π:2

D. π:4

 

Q. 43 ABCD is a square with AB = (x + 16) em and BC = (3x) em. The perimeter (in em) of the square is

A. 16

B. 24

C. 32

D. 96

 

Q. 44 The mean of 10 numbers is 0. If 72 and -12 are included in these numbers, the new mean will be

A. 0

B. 6

C. 5

D. 60

 

Q. 45 The circumference of the base of a right circular cylinder is 44 em and its height is 15 cm. The volume (in cubic centimeter) of·the cylinder is

A. 770

B. 1155

C. 1540

D. 2310

 

Q. 46 A class VII teacher wants to discuss the following problem in the class :

“A square is divided into four congruent rectangles. The perimeter of each rectangle is 40 units. What is the perimeter of given square ?”

Key Mathematical concepts required to solve this problem is

A. Area of square· and rectangle, perimeter of square and rectangle and definition of square and rectangle

B. Meaning of the terms square, rectangle, congruent, perimeter, etc.

C. Area of rectangle, perimeter of square

D. Formation of algebraic equation to solve the problem.

 

Q. 47 Algebra is introduced in the middle classes. According to Piaget’s’ theory of cognitive development, it is appropriate to introduce algebra at this stage as

A. the child is at sensor motor stage and can understand with the help of lots of manipulates.

B. the child is: at pre-operational stage and can understand abstract concepts.

C. the child is at concrete-operational stage and he can understand and conceptualize concrete experiences by creating ·logical structure.

D. the child is at formal operational stage and is fully mature to grasp the abstract concepts.

 

Q. 48 Mr. Sharma was assessing the students’ work on exponents. One of the response sheet was as follows :

a)2³ x 2⁵ = 2⁸

b)3² x 4² = 12⁴

c)3³ + 3⁵ = 1/3²

d)9³ + 18⁶ = (1/2)³

On the basis of this response sheet Mr. Sharma can make the following observations :

 

A. Child has understood the laws of exponents and can apply them well.

B. Child has understood the laws of exponents but has made clerical errors.

C. Child has understood the laws of exponents but has not practised the questions involving division of two numbers

D. Child has understood the law of exponents for the cases where the base is same and has missed the · concept for the case where the base is different.

 

Q. 49 According to Van Hiele level of Geometric thought the five levels of geometric understanding are visualization, analysis, informal deduction, formal deduction and rigour.

Students of class Vll are asked to classify the quadrilaterals according to their properties.

These students are at __ level of Van Hiele Geometric thought.

A. Visualization

B. Analysis

C. Informal Deduction

D. Formal Deduction·

 

Q. 50 A task assigned to the class Vlll student is as follows : An open box is to be made out of a metallic sheet of 50· em x 65 em. Length and breadth of the box is 30 .em and 15 em respectively. What is the possible height of the· box ? Also find the volume of this box. This task refer to

A. lower level cognitive demand as it requires the knowledge of formulae of volume of cuboid.

B. lower level of cognitive demand as there is no connection between concepts involved and procedure required.

C. higher level of cognitive demand is the problem can be solved by making diagrams and connections between many possible solutions

D. higher level of cognitive demand as it requires the use of conceptual understanding that underline the procedure to complete the task.

 

Q. 51 Mr. Nadeem gave the following task to his class, after completing the topic on lines and angles :

Speak for 2 minutes on the following figures using your knowledge about lines and and angles.This task is

A. Reflective in nature, can be used to pass time.

B. Exploratory in nature, can be used for summative assessments.

C. reflective as well can be used for formative assessment

D. communicating and can encourage mathematical talk in the classroom during free time

 

Q. 52 A child of class Vll defined the rectangle as follows : “Rectangle is a quadrilateral whose opposite sides are parallel and equal.” The definition reflects that the child

A. cannot recognize the shape.

B. do not know the correct properties of the shape.

C. knows the properties of the shape, but repeated some properties in definition.

D. knows some; properties, but missed some important to complete the definition.

 

Q. 53 A very common error observed in addition of linear expression is 5y+3=8y. This type of error is termed as

A. Clerical error

B. Conceptual error

C. Procedural error

D. Careless error

 

Q. 54 Read the following question from the class VI text-book : “Prabal deposited t 5,000 in a_ bank at the rate of 5% interest per annum. After 2 years he withdrew the money to purchase the study table for rs. 3,500. He deposited! the money left ·with him again at the rate of 5% interest per annum for another two years. How much amount will he receive after two years ?” What values can be inculcated in students through this question ?

A. Honesty

B. Habit of saving money and spending it wisely

C. Sincerity

D. Simplicity and helping others

 

Q. 55 Following is a problem from text-book of class VI: “Express the following statement through linear expression : Neha has 7 more toffees than Megha. If Megha has x toffees, how many toffees does Neha have ?” Which competence of Bloom’s cognitive domain is referred -in the above question ?

A. Knowledge

B. Comprehension

C. Analysis

D. Synthesis

 

Q. 56 If p = 3²⁰⁰⁰ + 3⁻²⁰⁰⁰ and q = 3²⁰⁰⁰ + 3⁻²⁰⁰⁰, then the value of p² – q² is

A. 1

B. 2

C. 3

D. 4

 

Q. 57 The square of 9 is divided by the cube root of 125. The remainder is

A. 1

B. 2

C. 3

D. 4

 

Q. 58 The value of √(16√(8)√(4)) is

A. 16

B. 8

C. 8∛2

D. 16√2

 

Q. 59 One-half of 1.2 x 10³⁰ is

A. 6.0 x 10³⁰

B. 6.0 x 10²⁹

C. 0.6 x 5³⁰

D. 1.2 x 10¹⁵

 

Q. 60 If 2/3, 23/30, 9/10, 11/15 and 4/5 are written in ascending order, then the fraction in the middle most

A. 23/30

B. 4/5

C. 2/3

D. 11/15

 

Q. 61 The steps required for the conversion of kitchen garbage into manure are given below in a jumbled form. t:

(A) Put the garbage in the pit.

(B) Cover the bottom of the pit with sand.

(C) Cover the pit loosely with grass or a gunny bag.

(D) Add worms.

The correct sequence of these steps is:

A. (B), (A), (C), (D)

B. (B), (C), (A), (D)

C. (B), (A), (D), (C)

D. (B), (D), (A), (C)

 

Q. 62 Two organisms are best friends and live together. One provides shelter, water and nutrients while the other prepares and provides food. Such an association of organisms is termed as

A. Autotrophy

B. Parasitism

C. Hetrotrophy

D. Symbiosis

 

Q. 63 In the alimentary canal. the swallowed· food moves downwards because of

A. the contraction of muscles in the wall of food pipe.

B. the flow of fluid material taken

C. gravitational pull.

D. force provided by the muscular tongue.

 

Q. 64 Frogs and earthworms breathe through their skin because of which the skin of both the organisms is

A. dry and rough

B. dry and slimy

C. moist and rough

D. moist and slimy

 

Q. 65 While going for a picnic a student noted the reading on the odometer on the bus after every 10 minutes till the end of the journey. Later on he recorded the reading in a table shown below:

The average speed of the bus in the entire journey in metres per second was

A. 15

B. 18

C. 30

D. 54

 

Q. 66 You are provided with a concave mirror, a concave lens, a convex mirror and a convex lens. To obtain an enlarged image of an object you can use either

A. concave lens or .convex lens

B. concave mirror or convex mirror

C. concave mirror or concave lens

D. concave mirror or convex lens

 

Q. 67 Select from the following a set of Kharif Crops :

A. Cotton, paddy, pea, linseed

B. Paddy, maize, cotton, soyabean

C. Gram, mustard, groundnut, wheat

D. Maize, paddy, linseed, soyabean

 

Q. 68 Study the following statements about the effects of weeds on the crop plants:

(A) They help crop plants to grow healthily.

(B) They interfere in harvesting.

(C) They affect plant growth.

(D) They compete with crop plants for water, nutrients, space and light. The correct statements are :

A. (C) and (D) only

B. (D) only

C. (B), (C) and (D)

D. (A), (B) and (D)

 

Q. 69 When we add aluminium foil to freshly prepared sodium hydroxide solution a gas is produced. Which of the following correctly states the property of this gas ?

A. Colourless and odourless gas which extinguishes a burning match stick.

B. Colourless and odourless gas which promotes burning of a candle.

C. Brown coloured pungent smelling gas.

D. Colourless and odourless gas which produces a ‘pop’ sound. when a burning match stick is brought near it.

 

Q. 70 Aqueous solution of which of the following oxides will change the colour of blue litmus to red ?

A. Copper oxide

B. Iron oxide

C. Magnesium oxide

D. Sulphur dioxide

 

Q. 71 Which of the following· is a pair of exhaustible natural resources ?

A. Coal and soil

B. Petroleum and water

C. Minerals and wildlife

D. Natural gas and sun-light

 

Q. 72 Which of the following statements is true about endemic species ?

A. They are not affected by the destruction of their habitat.

B. They are found only in zoos and botanical gardens.

C. They are found exclusively in specific habitat.

D. Endemic species can never become endangered.

 

Q. 73 Which of the following pairs is related to the inheritance of characters ?

A. Chromosomes and genes

B. Chromosomes and mitochondria

C. Cell membrane and cell wall

D. Cell membrane and chloroplast

 

Q. 74 Out of the different combinations of terms given below, the correct combination of terms with reference to an animal cell is

A. Nucleus, plastid, cell membrane, cell wall

B. Nucleus, chromosome, ribosome, cell wall

C. Cell membrane, chromosome, ribosome, mitochondria

D. Cell membrane, ribosome, chloroplast, mitochondria

 

Q. 75 Consider the following sets of reproductive terms

(A) Sperm, oviduct, egg, uterus

(B) Ovulation, egg, oviduct, uterus

(C) Sperm, testis, sperm duct, penis

(D) Menstruation, egg,. oviduct, uterus·

The sets of correct combination are :

 

A. (A), (B) and (C)

B. (B), (C) and (D)

C. (C), (D) and (A)

D. (A), (B) and (D)

 

Q. 76 Which of the following elements must be available in water for the metamorphosis of tadpoles ?

A. Chlorine

B. Bromine

C. Sulphur

D. Iodine

 

Q. 77 Which one of the following is not the objective of teaching of Science at upper primary stage ?

A. Developing questJ.onmg and enquiring skills

B. Acquiring technological skills

C. Acquiring process skills

D. Acquiring scientific literacy

 

Q. 78 National Curriculum Framework (NCF) strongly recommends that Science education at upper primary stage should

A. prepare students for competitive examinations.

B. help students to be emotionally balanced.

C. help students to acquire computational skills.

D. follow constructivist approach for teaching and learning of the subject.

 

Q. 79 ‘Cognitive validity’ of Science curriculum at upper primary stage requires that it should 

A. be age appropriate and within the reach of students’ understanding level.

B. convey scientifically correct content.

C. nurture the natural curiosity and creativity of the learners.

D. enable the students to appreciate how · the concepts of Science evolve with time.

 

Q. 80 Which one of the following does not reflect the personality attribute of a person having scientific temper ?

A. Seeking evidence·

B. Biased opinion

C. Rational thinking

D. Open mindedness

 

Q. 81 NCERT Science textbooks for upper primary classes include large number of daily life related questions which have been left unanswered. This has been done so that

A. teachers have: a good pool of questions for assessment purpose.

B. students can send these questions to scientists to get the answer.

C. teachers can use these questions for home assignment.

D. students can seek answer to these questions by exploring different resource

 

Q. 82 While teaching the correct method of reading a clinical thermometer to class VIII students, Neha mentions the following necessary precautions to be taken:

Which one of the above precautions has been mentioned wrongly by the teacher?

A. Thermometer should be washed with hot water. before and after use.

B. Do not hold the thermometer by the bulb while taking the reading.

C. Ensure that before use, the mercury level in the thermometer is below 35 °C.

D. Read the thermometer keeping the level of mercury along the line of sight.

 

Q. 83 The section on ‘activities and projects’ included in the exercises· of NCERT Science textbooks for class Vlll primarily aims at

A. enhancing in depth understanding of the basic concepts.

B. keeping the students engaged during vacations.

C. assessing the students on practical skills.

D. providing opportunity to students for extended learning.

 

Q. 84 Major. objective of organisation of . Science Exhibitions is to

A. grade students on practical skills.

B. provide opportunity to students to compete with others.

C. provide opportunity to students to showcase their creative ideas.

D. provide opportunity to students to enhance their academic performance.

 

Q. 85 Four candidates appearing in an interview for the post of science teacher were asked to give a demonstration lesson to class VIII students on the topic ‘Pressure exerted by liquids and gases’. Following different approaches were followed by different candidates :

Which one of the above approaches will be most effective for teaching of the topic?

A. Detailed explanation of related concepts with the help of diagram on the blackboard.

B. Use of charts for _explanation of different concepts.

C. Organisation of hands-on student activities followed up with discussions.

D. Greater focus on . classroom questions during the lecture.

 

Q. 86 The technique of ‘classroom questioning’ in teaching of Science can be more effectively used for

A. ensuring levels of learning

B. developing problem solving skills

C. maintaining discipline in the Class

D. promoting creativity and innovativeness

 

Q. 87 Given below are the steps to test the presence of proteins in a food item.

These steps are not in correct sequence.

(A) Take a small quantity of food item in a test tube, add 10 drops of water to it and shake it.

(B) Make the paste or powder of the food to be tested.

(C) Add 10 drops of caustic soda solution to the test tube and shake well.

(D) Add 2 drops of copper sulphate

to it. The correct sequence of these steps is

 

A. (A), (B), (D), (C)

B. (B), (A), (D), (C)

C. (B), (A), (C), (D)

D. (D), (B), (A}, (C)

 

Q. 88 Which of the following terms constitute the female part of a flower ?

A. Stigma, ovary and stamen

B. Stigma, ovary and style

C. Stamen, ovary and style

D. Stamen, petals and sepals

 

Q. 89 How many muscles work together to move a bone?

A. two

B. four

C. six

D. number is not fixed

 

Q. 90 Choose the set that represents only the biotic component of a habitat.

A. Sand, turtle, crab, rocks

B. Insects, frog, fish, aquatic plants

C. Tiger, deer, grass, soil

D. Insects, water, aquatic plants, fish

 

Questions: 91 – 99

Renowned educationist Sir Tim Brighouse, observed that an outstanding school has four factors that are visible. “Teachers talk about teaching, teachers observe each other’s practice, teachers plan, organize and evaluate their work together rather than separately, and that teachers teach each other.” He continues : “One of the reasons I like that is that you can immediately see ways in which you could make it more likely that teachers talk about teaching.” Sir Tim then encouraged schools to focus on activities .that were low effort but high impact, describing them as “butterflies”. Some . examples he gave included rotating staff meetings around different classrooms with the host, at the start, describing the room layout and displays, or discussing other teaching techniques ·and approaches. With modem technology teachers could observe their own. lessons and then. when viewing them back, decide whether they want to share them with a mentor. The role of mentoring was vital and suggested that more schools could send teachers out in small groups to learn from colleagues in other schools. He said : “If this were widespread practice, if people were to attend to their butterflies, the outcome in terms of teacher morale and teacher satisfaction would be positive. We all agree that professional development is the vital ingredient”. 

 

Q. 91 In the mode suggested by Sir Tim, teachers may self-evaluate and self reflect

A. using technology

B. without technology

C. through a students’ surve

D. interviewing each other

 

Q. 92 ‘Teachers talk about teaching’ means that they

A. make some suggestions

B. discuss their own practices

C. criticize one another

D. freely change opinions

 

Q. 93 ‘Low effort but high impact’ in this context implies that schools

A. pay teachers a lower salary

B. extract more work for the same pay

C. decrease the workload and salary

D. create opportunities within the system for development

 

Q. 94 In this extract, it is observed that technology supports teachers to

A. improve students

B. conduct meetings for teachers

C. follow-up/remediation activities for students.

D. self-diagnose their practices

 

Q. 95 Here, ‘visible’ means

A. seen

B. obvious

C. appealing

D. bright

 

Q. 96 ‘Rotating staff. meetings in the classrooms’ permits teachers to

A. be informal with each other

B. miss some of them

C. share their own practices with others

D. keep busy all the time

 

Q. 97 A synonym for the word, ‘counselling’, from.the passage is

A. describing

B. discussing

C. mentoring

D. teaching

 

Q. 98 The talk by Sir Tim is about the

A. teachers who dress like butterflies.

B. visiting schools to socialise.

C. knowledge teachers gain for job growth.

D. schools who control their teachers.

 

Q. 99 “Butterflies” here refer to

A. the dress code

B. a practice of staff interaction

C. changing schools

D. going to classes in rotation

 

Questions: 100 – 105

Do you know how much you mean to me·?

As you grow into what you will be.

You came from within, from just beneath my heart

It’s there you’ll always be though your own life will now start.

You’re growing so fast it sends me a whirl,

With misty eyes I ask, Where’s my little girl?

I know sometimes to you I seem harsh and so unfair,

But one day you will see, I taught you well because I care.

The next few years will so quickly fly,

With laughter and joy, mixed with a few tears to cry.

As you begin your growth to womanhood, this fact you must know,

You’ll always be my source of pride, no matter where you go.

You must stand up tall and proud, within you feel no fear, .

For all you dreams and goals, sit before you very near.

With God’s love in your heart and the world by its tail,

You’ll always be my winner, and victory will prevail.

For you this poem was written, with help from above,

To tell you in a rhythm of your Mother’s heartfelt love !

 

Q. 100 ‘Do you know how much you mean to me ?’ is a question.

A. rhetorical

B. restrictive

C. convergent

D. divergent

 

Q. 101 An antonym for the word ‘harsh’ is

A. severe

B. mild

C. grim

D. clashing

 

Q. 102 In the expression ‘It sends me a whirl’, ‘it’ refers to

A. travelling far

B. growing up

C. new experiences

D. the real world

 

Q. 103 To ‘stand up tall’ is

A. growing up healthy .

B. be tail like the boys

C. being fearless

D. getting ambitious

 

Q. 104 The phrase ‘the world by its tail’ means to

A. be a good follower

B. overcome challenges

C. face one’s enemies

D. to avoid challenges

 

Q. 105 The poem addresses a

A. friend

B. daughter

C. mother

D. girl

 

Q. 106 A person sitting behind you in a cinema starts talking on the mobile phone and you want to tell him/her to stop. Choose how you will make the request.

A. “Stop talking so loudly !”

B. “Please don’t use the mobile phone inside the theatre.”

C. “can’t you not use the mobile phone; please ?”

D. “Could you possibly stop using the mobile phone here ?”

 

Q. 107 Example for homonym would be

A. aisle/isle

B. beer/bear

C. stale/stall

D. stock/stoke

 

Q. 108 Which is a lexical word?

A. love

B. is

C. the

D. might

 

Q. 109 While learning about the passive voice form, students learn about

A. use of by

B. position of verb

C. position of nouns

D. use of verb forms

Q. 110 Students can leave the school premises at 12.30 pm. Students ought to leave the school premises at 12.30 pm. The two given statements can be differentiated by drawing students’·attention to the

A. differences in the arrangement of words

B. meaning conveyed by the models/verb modifiers

C. the roles of the subject and object in both sentences

D. absence of change in the verb form ·

 

Q. 111 Which learning domain constitutes higher order thinking ?

A. remembering

B. understanding

C. application

D. evaluation

 

Q. 112 The process of word formation consists of

A. compounding and affixes

B. opposites and meaning

C. verbs and noun

D. using synonyms or euphemisms

 

Q. 113 When reading, to ‘decode’ means to

A. an action used in ICT

B. solving a complex puzzle

C. to analyse and understand

D. understanding a foreign language

 

Q. 114 Speaker 1 :’We had an enjoyable holiday this winter.

Speaker 2 Where did you go ?

Speaker 1 Where ?

Speaker 2 Yes, which place did you visit?

During the assessment of students’ speaking-listening skits; mark/s would be deducted during this exchange for

 

A. Speaker I

B. Speaker II

C. Both

D. Neither

 

Q. 115 While writing, ‘ellipses’ is a

A. phrasal verbs

B. semantic linker

C. adjectival form

D. prepositional phrase

 

Q. 116 If a longer piece of writing is brief, complete, in the third person, without digressions and emotional overtones and logically arranged, it is a

A. classified advertisement

B. memorandum

C. report

D. newspaper article

 

Q. 117 he politician had been making promises long before election time. The statement is in the tense.

A. past continuous

B. present perfect

C. present perfect continuous

D. past perfect continuous

 

Q. 118 ‘Gender sensitization’ in the school curriculum implies

A. children should be differentiated as boys and girls

B. sex education from primary school onwards

C. respectful approach towards defining gender roles

D. promote co-education in high school.

 

Q. 119 Identify where the collective form is an error:

A. host of angels

B. congress of baboon

C. clutch of ducks

D. shoal of fish

 

Q. 120 An example of a question to ‘funnel’ or restrict a respondent’s answer is

A. “What do you think of the weather?”

B. “How many books are there ?”

C. “Tell me about your most recent holiday.”

D. “What are your goals ?”

 

Questions: 121 – 129

Raja Ravi Varma was the Indian King and painter whose paintings . brought a momentous turn in Indian art. His works on great Indian epics Ramayana and Mahabharata brought the omnipresent deities to the surroundings of earthy world. This showed , excellent fusion of Indian traditional art with European realism. These paintings influenced future generation artists and also influenced the literature and films. His representation of mythological characters has become a part of the Indian imagination of the Classics. His style is criticized for being too gaudy and sentimental. Ravi Varma was born on April.29th, 1848 in Kilimanoor Palace in Kerala. Ravi Verma was brought up in an environment of “art and culture. At the age of seven he started painting the figures of animals, acts and scenes from daily life on the wall with charcoal. As he grew up, he was exposed to the famous paintings of Italian painters. Here he was using indigenous paints made from leaves, flowers. He enhanced his creativity by listening to the music of veterans, watching Kathakali, a folk dance form, going through the manuscripts preserve in ancient families and listening to the artistic interpretation of the epics. Raja Ravi Verma is most remembered for his paintings of beautifully sari-clad worried, who were depicted as graceful and shapely. 

 

Q. 121 The themes of Ravi Varma’s famous paintings were

A. deities

B. animals and habitats

C. natural scenery

D. female figures

 

Q. 122 He was especially able to access historical documents in the possession of

A. national museums that curate them

B. certain individuals

C. families who inherited them

D. his family members in Kilimanoor

 

Q. 123 As he matured in his craft, Ravi Verma’s·skills were influenced by

A. Italian artists

B. Indian cinema

C. Ancient manuscripts

D. None of these

 

Q. 124 ‘His style is criticized for being too gaudy and sentimental’ means that his work was characterized by

A. pale colours and sad atmosphere

B. unrealistic images

C. lacking intellectual and emotional depth·

D. strong colours and emotionally appealing

 

Q. 125 In the extract, ‘artistic interpretation of the epics’ means he was interested in

A. popular writing of his time.

B. unique rendering of old mythologies.

C. standard interpretation of the Indian epics.

D. unusual and rare myths found in legends.

 

Q. 126 Find a word in the passage which is the opposite of ‘minimized’

A. influenced

B. criticized

C. exposed

D. enhanced

 

Q. 127 The article is alan

A. fiction

B. essay

C. biography

D. autobiography

 

Q. 128 The focus is on the subject’s association with

A. sculpture

B. painting

C. music

D. dance (Kathakali)

 

Q. 129 A synonym for ‘omnipresent’ is

A. conspicuous

B. universal

C. partly invisible

D. magnificent

 

Questions: 130 – 135

The scene presented by a community of bees is the more astonishing, the more we become acquainted with its details. Each hive is a commonwealth, of which the queen is nominally the head, receiving the greatest honour and care from her industrious subjects. With a greater wisdom than can be claimed by men, these creatures allow no disputes about the succession to the throne to induce them to injure each other; but they require the parties themselves individually to settle the quarrel between each other, without prolonged interference with the duties of the hive. Indeed, they may be said with truth to have adopted the advice : “Let those who make the quarrels, be the only ones to fight.” Only one queen is permitted to hold office in the community at a time; but while her claims are undisputed, she is treated with singular respect and affection. Indeed, her presence, and the prospect of a future generation, appear the chief motives of the insects to exert themselves. )

 

Q. 130 The writer’s observation that the bees ‘settle the quarrel between each other, without prolonged interference with the duties of the hive’ suggests that he/she is

A. pointing a bee character

B. observing the traits of worker bees

C. appreciating the queen bee

D. condemning the behaviour of ill tempered human beings

 

Q. 131 ‘The prospect of a future generation,suggests that the writer is in his/her outlook.

A. ambivalent

B. nationalistic

C. sympathetic

D. appreciative

 

Q. 132 ‘The parties’ in this context means

A. participants

B. political organisation

C. individuals

D. bees

 

Q. 133 The queen bee has a position which gives her from her subordinates.

A. disregard but affection

B. aggressive defensiveness

C. exceptional reverence and devotion

D. caring support and indulgence

 

Q. 134 ‘To exert themselves’ means the bees are

A. having influence

B. quite aggressive

C. busy toiling

D. very exhausted

 

Q. 135 ‘Each hive is a commonwealth ‘ means the hive is

A. over-crowded

B. has a queen

C. self-governing

D. without a leader

 

Q. 136 Compounding is

A. stringing together older words like the formation of earthquake from earth and quake.

B. removing seeming affixes from existing words, such as forming edit from editor.

C. joining parts of two or more older words, such as forming smog, which comes from smoke and fog.

D. forming new words from existing ones by adding suffixes to them, like shame + less + ness -+ shamelessness

 

Q. 137 Constructivism is a theory where students

A. study a variety of dissimilar samples and draw a well founded conclusion.

B. form their own understanding and knowledge of the world, through experiencing things and reflecting on those experiences.

C. are facilitated by the teacher and use a variety of media to research and create their own theories.

D. construct their own learning aids, thereby · gaining hands-on experience.

 

Q. 138 E-learning refers to

A. acquisition of the mother tongue

B. learning English language as the first language.

C. use of electronic media and information and communication technologies

D. a language course for foreign languages

 

Q. 139 Examples of irregular verbs are

A. break – broke

B. train – trainee

C. walk – walked

D. happy – happier

 

Q. 140 A subject-centered approach is where learning experiences are organized around 

A. students’ interests

B. teacher’s subject competence

C. course content

D. educational technology

 

Q. 141 Standardized assessment means ___ assessment.

A. formative

B. summative

C. frequent

D. alternative

 

Q. 142 In the word ‘flower’, the phonetic transcription is

A. /fleur/

B. /fl::eur/

C. /flauer/

D. /flour/

 

Q. 143 The Gathering skill while reading is

A. note-taking

B. note-making

C. puzzling out

D. analysis

 

Q. 144 Multiple choice items consist of a

A. set of questions

B. options to choose questions

C. sequence of grammatical errors

D. stem and a set of options

 

Q. 145 Abilities for adaptive and positive behaviour that enable students to dell effectively with everyday· demands and challenges are

A. multiple intelligences

B. learning domains

C. life skills

D. learning methods

 

Q. 146 A self-contained unit of a discourse in writing dealing with a particular point or idea is

A. an essay

B. a poem

C. a paragraph

D. a statement

 

Q. 147 The following is a conditional sentence

A. I have to go to work.

B. lf the sea is stormy, the waves are high.

C. Shut the door.

D. It’s a sunny day, isn’t it?

 

Q. 148 Communication technology that enable sharing of educational resources within an institution

A. internet

B. intranet

C. telephone

D. audio-video cassette

 

Q. 149 Encouraging the student to love the world and to imagine a peaceful future; and caring for the student and encouraging the student to care for others is education.

A. distance

B. academic

C. values

D. vocational

 

Q. 150 A communication technique that requires the listener to feed back what they have heard in their own words, to confirm the understanding of both parties, is listening.

A. active

B. appreciative

C. informative

D. passive

 

Answer Sheet 
Question 1 2 3 4 5 6 7 8 9 10
Answer A D C A D B A B A C
Question 11 12 13 14 15 16 17 18 19 20
Answer B C C A C C A B D A
Question 21 22 23 24 25 26 27 28 29 30
Answer B A A C D B B C B B
Question 31 32 33 34 35 36 37 38 39 40
Answer C C A C D A B A C A
Question 41 42 43 44 45 46 47 48 49 50
Answer A D D B D B C C B D
Question 51 52 53 54 55 56 57 58 59 60
Answer C D B B B D A B B A
Question 61 62 63 64 65 66 67 68 69 70
Answer C D A D A D B C D D
Question 71 72 73 74 75 76 77 78 79 80
Answer C C A C B D B D A B
Question 81 82 83 84 85 86 87 88 89 90
Answer D A D C C A B B A B
Question 91 92 93 94 95 96 97 98 99 100
Answer A B D D B C B C B D
Question  101 102 103 104 105 106 107 108 109 110
Answer B B C B B B A A D B
Question  111 112 113 114 115 116 117 118 119 120
Answer D A C A B C D C C C
Question 121 122 123 124 125 126 127 128 129 130
Answer D C A D B D C B B D
Question 131 132 133 134 135 136 137 138 139 140
Answer B A C C C A B C A C
Question 141 142 143 144 145 146 147 148 149 150
Answer B B B D C C B B C A

CTET July 2013 Paper-I Previous Year Paper

CTET July 2013 Paper-I


Q. 1 The following three aspects of intelligence are dealt by Sternberg’s traiarchic theory except

A. componential

B. social

C. experiential

D. contextual

 

Q. 2 Howard Gardner’s theory of multiple intelligences emphasizes

A. general intelligence

B. common abilities required in school

C. the unique abilities of each individual

D. conditioning skills in students

 

Q. 3 The sounds th, ph, ch are

A. Morphemes

B. Graphemes

C. Lexemes

D. Phonemes

 

Q. 4 In order to avoid gender stereotyping in class, a teacher should

A. try to put both boys and girls in non-traditional roles.

B. appreciate students’ good work by saying ‘good girl’ or ‘good boy’

C. discourage girl from taking part in wrestling.

D. encourage boys to take risk and be bold

 

Q. 5 Schools should cater to Individual differences to

A. narrow the gap between individual students.

B. even out abilities and performance of students.

C. understand why students are able or unable to learn.

D. make individual students feel exclusive.

 

Q. 6 What kind of support can a school provide to address the individual differences in students ?

A. Follow a child-centered curriculum and provide multiple learning opportunities to students

B. Apply every possible measure to remove the individual differences in students

C. Refer slow learners to special schools

D. Follow same level of curriculum for all students

 

Q. 7 Continuous and Comprehensive Evaluation emphasizes

A. continuous testing on a comprehensive scale to ensure learning.

B. how learning can be observed, . recorded and improved upon.

C. fine-tuning of tests with the teaching.

D. redundancy of the Board examination.

 

Q. 8 School Based Assessment .

A. Dilutes the accountability of Boards of Education.

B. ·Hinders achieving Universal National Standards.

C. Helps all students learn more through diagnosis

D. Makes stude:nts and teachers nonserious and casual.

 

Q. 9 “Readiness for learning” refers to

A. general ability level of students

B. present cognitive level of students in the learning continuum

C. satisfying nature of the act of learning

D. Thorndike’s Law of Readiness

 

Q. 10 A teacher has some physically challenged children in her class. Which of the following would be appropriate for her to say ?

A. Wheel-chaired bound children may take help of their peers in going to hall.

B. Physically inconvenienced children may do an alternative activity in the: classroom.

C. Mohan why don’t you use your crutches to go to the playground.

D. Polio afflicted children will now present a song.

 

Q. 11 Learning disabilities may occur due to all of the following except.

A. Cerebral dysfunction

B. Emotional disturbance

C. Behavioural disturbance

D. Cultural factors

 

Q. 12 An inClusive school.

A. Is committed to· improve the learning outcomes of all students irrespective of their capabilities

B. Differentiate between students and sets less challenging achievement targets for specially abled children

C. Committed particularly to improve the learning outcomes of specially abled students

D. Decides learning heeds of students according to their disability

 

Q. 13 Gifted students

A. Need support not ordinarily provided by the school

B. Can manage their studies without a teacher

C. Can be good models for other students

D. Cannot be learning disabled

 

Q. 14 Giftedness is due to

A. Genetic makeup

B. Environmental motivation

C. Combination of(l) and (2)

D. Psychosocial factors

 

Q. 15 Which of the following is appropriate for environment conducive to thinking and learning in children ?

A. Passive listening for long periods of time

B. Home assignments given frequently

C. Individual tasks done by the. learners

D. allowing students to take some decisions about what to learn and how to learn

 

Q. 16 Learning Disability in motor skills is called

A. Dyspraxia

B. Dyscalculia

C. Dyslexia

D. Dysphasia

 

Q. 17 Learning Disability

A. is a stable state

B. is a variable state

C. need not impair functioning

D. does not improve with appropriate input

 

Q. 18 The following are the steps in the process of problem solving except

A. Identification of a problem

B. Breaking down the problem into smaller parts

C. Explore possible strategies

D. Anticipate outcomes

 

Q. 19 A teacher should

A. treat errors committed by students as blunders and take serious note of each error

B. measure success as the number of times students avoid making mistakes

C. not correct students while they’re trying to communicate ideas

D. focus more provide a knowledge on lecturing and foundation for

 

Q. 20 Seema is desperate to score A+ grade in an examination. As she enters the examination hall and the examination begins, she becomes extremely nervous. Her feet go cold, her heart starts pounding and she is unable to answer properly. The primary reason for this is that

A. she may not be very confident about her preparation

B. she may be thinking excessively about the . result of this examination

C. invigilator teacher on duty may be her class teacher and she is of very strict nature

D. she may not be able to deal with sudden emotional outburst

 

Q. 21 Which of the following cognitive verbs are used to analyse the information given? 

A. IdentifY

B. Differentiate

C. Classify

D. Describe

 

Q. 22 Rajesh is a voracious reader. Apart from studying his course · books, he often goes to library and reads books on diverse topics. Rajesh does his project even in the lunch break. He does not need prompting by his teachers or parents to study for tests and seems to truly enjoy learning. He can be best described as a(n) __ _

A. fact centred learner

B. teacher motivated learner

C. assessment-centered learner

D. intrinsically motivated learner

 

Q. 23 Children in pre-primary get satisfaction room being allowed to discover. They become distressed, when they are discouraged. They do so due to their motivation to

A. reduce their ignorance

B. affiliate with the class

C. create disorder in the class

D. exercise their power

 

Q. 24 Understanding Human Growth and Development enables a teacher to

A. gain control of learners’ emotions while teaching

B. be clear about teaching diverse learners.

C. tell students how they can improve their lives

D. practice her ·teaching m an unbiased way.

 

Q. 25 Which one of the following is true ?

A. Development and learning are unaffected by socio-cultural contexts.

B. Students learn only in a certain way

C. Play is significant for cognition and social competence

D. Questioning by teacher constrains cognitive development.

 

Q. 26 Which one of the following is true about the role of heredity and environment in the development of a child?

A. The relative contributions of peers and genes are not additive.

B. Heredity and environment do not operate together.

C. Propensity IS related to environment while actual development requires heredity.

D. Both heredity and environment contribute 50% each m the development of a child. 

 

Q. 27 Socialization is

A. Rapport between teacher and taught

B. Process of modernization of society

C. Adaptation of social norms

D. Change in social norms

 

Q. 28 A PT teacher wants her students to improve fielding in the game of cricket. Which one of the following strategies will best help his students achieve that goal ?

A. Tell students how important it is for them to learn to field.

B. Explain the logic behind good fielding and rate of success.

C. Demonstrate fielding while students observe

D. Give students a lot of practice in fielding

 

Q. 29 A teacher wishes to help her students to appreciate multiple views of a situation. She provides her students multiple opportunities to debate on this situation in different groups. According to Vygotsky’s perspective, her students will various· views and develop multiple perspectives of the situation on their own.

A. internalize

B. construct

C. operationalize

D. rationalize

 

Q. 30 Sita has learned to eat rice and dal with her hand. When she is given dal and rice, she mixes rice and dal and starts ·eating. She has eating rice and dal into her schema for doing things. 

A. Accommodated

B. Assimilated

C. Appropriated

D. Initiated

 

Q. 31 Perimeter of a square is 24 em and length of a rectangle is 8 em. If the perimeters of the square and the rectangle are equal, then the area (in square em) of the rectangle is

A. 16

B. 24

C. 32

D. 64

 

Q. 32 The difference of the place value and the face value of the number 3 in 12345 is

A. 0

B. 295

C. 297

D. 305

 

Q. 33 Which one of the following is not correct?

A. 56.7 kilogram= 5670 grams

B. A ·cube has six faces

C. One millimetre= 0.1 em

D. 0.10 is same as 0.1

 

Q. 34 The speed of a boat in a river is 20 kmper hour and the speed of another boat is 23 km per hour. They travel in the same direction from the same place at the same time. The distance between the boats after three and half hours is

A. 10 km

B. 10.5 km

C. 11 km

D. 11.5 km

 

Q. 35 When 90707 is divided by 9, the remainder is

A. 3

B. 5

C. 6

D. 7

 

Q. 36 When a fresh fish is dried it becomes 1/3 of its weight. Sunita buys 1500 kg fresh fish for rs. 25 per kg and sell them, when dried, for rs. 80 per kg. How much does she earn ?

A. Rs.2,500

B. Rs.2,700

C. Rs.3,000

D. Rs.3,700

 

Q. 37 Look at the following pattern :

(9-1)÷8= 1

(98-2)÷8=12

(987 – 3) ÷ 8 = 123

(9876- 4) ÷ 8 = 1234

According to this pattern

(987654- 6) ÷ 8 =

 

A. 12345

B. 123456

C. 123465

D. 123467

 

Q. 38 750 ml juice is. fiUed in one bottle and six such bottles are packed in one carton. The number of cartons needed for 450 litres of juice is

A. 75

B. 80

C. 90

D. 100

 

Q. 39 Internal length, breadth and depth of a (rectangular) box are 4 em, 3 em and 2 em respectively. How many_such boxes are needed to pack 8664 centimetre cubes ?

A. 351

B. 361

C. 391

D. 722

 

Q. 40 “Write the equivalent fraction of 3 .” The above question asked to students of Class IV refers to

A. lower-level demand task as it requires procedural skills only.

B. lower-level demand task as it is based on memorization only.

C. higher-level demand task as it is based on procedure with connection.

D. higher-level demand task as it is based on procedure without connection

 

Q. 41 Students often make a mistake in comparing the decimal numbers. For example 0.50 is larger than 0.5. The most probable reason for this error is

A. lack of practice of these tyrpes of questions in the class.

B. lack of concrete experience of representation of decimal number on number line.

C. careless attempt by the students.

D. misconception regarding the significance · of zero in ordering decimal.

 

Q. 42 A teacher prompts the students to prepare Mathematical journal with the theme

“Application of Mathematics in Daily life”. This activity is 

A. to test tlhe students understanding of Mathematical concepts.

B. to provide opportunity to students share their ideas and knowledge.

C. to help students to sense of Mathematics.

D. to help students to connect Mathematical concepts and its applications and to share their knowledge and ideas.

 

Q. 43 According to Van Hiele level of geometric thought, the five )evels are – visualization, analysis, informal deduction, formal deduction arid rigour. Some polygons are given to a child of Class III for sorting.

He classified tbe polygons on the basis of the number of sides. This child is at ________ level of Van-Hiele Geometrical tbought

A. Visualization

B. ,’Analysis

C. lnformal deduction

D. Formal deduction

 

Q. 44 A child displays difficulty in differentiating between numbers, operations and symbols, two clock hands, different coins etc. This implies that the specific barrier affecting his. Learning is

A. poor verbal, visual, auditory and working memory.

B. poor visual processing ability i.e. visual discrimination, spatial organization and visual coordination

C. poor language processing ability. i.e. expression, vocabulary and auditory processing.

D. ·poor motor skills, reading and writing skills.

 

Q. 45 NCF 2005 emphasises on Constructivist Approach of learning as it focuses on

A. memorization of definitions and formulae

B. submission of regular homework.

C. active participation of Ieamer through engaging activities

D. effective lecture and instructions by teacher.

 

Q. 46 Following are array diagram bindis to represent 15.

The way of representing 15 or any . other number in the above manner can be used to

teach concept of

A. area and commutative,property

B. commutative property of multiplication, identification of prime and composite numbers, area of rectangle

C. representation of a number as product of two numbers, commutative property of multiplication, multiplicative identity, identification of prime · and composite numbers, area of rectangle using units quantity

D. representation of a number as product of two numbers, commutative property of multiplic:ation, multiplication identity, identification of prime and composite numbers.

 

Q. 47 Which of the following questions is open-ended ?

A. Write the numbers 25, 7I, I9, 9, 8, I7, 85 in ascending order.

B. Which is more ? 1/3 or 7/5

C. Write any four number greater than 2.7

D. What is 7 more than 2/7 ?

 

Q. 48 The most appropriate tool to expose the students of class II to plane figures, its vertices and edges is

A. Geo-Board

B. Nets of 3D solids

C. Cubes

D. Black-board surface

 

Q. 49 Following is a problem from text book of class V: “There are 4 poles of measure 105 cm, 115 cm, 150 cm and 235 cm respectively. If they have to be cut into pieces of equal length, what is the maximum length of each piece ?”

This question is asked to

A. test knowledge of factors and multiples

B. check the skill of finding HCF

C. enhance problem solving skills using learnt concepts

D. give practice of word problems based on HCF and LCM

 

Q. 50 Following is a problem from text book of class III :

“Which mathematical operation will be used to solve the following problem . ? A milkman sold I410 litres of milk in 10 days. How many litres of milk did he sell in a day ?”

Which competence of Bloom’s cognitive domain is referred in the above question ?

A. Knowledge

B. Comprehension

C. Analysis

D. Synthesis

 

Q. 51 Rashid is studying in class V. He can classify various types of triangles in different categories but has difficulty in understanding the abstract proof for the sum of three angles in a triangle to be always 180. According to Piaget Cognitive Theory Rashid is · at 

A. Concrete operational stage

B. Formal operational stage

C. Sensorimotor stage

D. Pre-operational stage

 

Q. 52 According to NCF 2005

“Developing children’s abilities for mathematization is the main goal of mathematics

education. The narrow aim of school mathematics is io develop ‘useful’ capabilities.”

Here mathematization refers to develop child’s abilities

A. In performing all number operations efficiently including of finding square root and cube root

B. To formulate Geometry and independently. Theorems of their proofs

C. To translate word problems into linear equations.

D. To develop the child’s resources to think and reason mathematically, to pursue assumptions to their logical conclusion and to handle abstraction.

 

Q. 53 The highlights of a good textbook are that

A. They contain numerous exercises to give rigorous practice.

B. All concepts can be introduced through situations.

C. Only solved examples are included.

D. They must” be thick and heavy.

 

A. A and B

B. C and D

C. A and C

D. B and D

 

Q. 54 NCF 2005 emphasises that

A. Succeeding m Mathematics should be mandatory for every child.

B. Students should be tested first for their logico-mathematical ability.

C. Maths curriculum shall be separate for low achievers.

D. Maths shall be taught to selective students.

 

Q. 55 The difference between the smallest common multiple and biggest common factor of 5, 10 and 35 is

A. 30

B. 35

C. 65

D. 75

 

Q. 56 The number of factors of 105 is

A. 3

B. 4

C. 6

D. 8

 

Q. 57 If the time now is 2.17 P.M., what will be. the time 11 hours and 59 minutes from now? 

A. 11:57 A.M.

B. 9 : 59 A.M.

C. 2: 16A.M

D. 2.17A.M.

 

Q. 58 Number of degrees in three and one and half right angeles is :

A. 285

B. 295

C. 305

D. 315

 

Q. 59 11 ones + 11 tens +11 hundred equals

A. 144

B. 1221

C. 12321

D. 111111

 

Q. 60 The sum of five hundred nine and three thousand twenty eight is:

A. 3537

B. 3087

C. 837

D. 387

 

Q. 61 The name of the scientist· who first peeped into a mosquito stomach and proved that mosquitoes spread malaria and for his research was awarded Nobel Prize in medicine in December 1902 is

A. Charles Darwin

B. Gregor Mendel

C. George Mistral

D. Ronald Ross

 

Q. 62 Rajat said to his friend, “I cannot play because I am down with fever. I pass through a cycle of shivering, fever and headache and finally sweating. .After my blood test doctor prescribed me a bitter medicine.” Rajat . might be suffering from

A. Typhoid

B. Diarrhoea

C. Cholera

D. Malaria

 

Q. 63 Who was AI-Biruni ?

A. A Qutabshahi Sultan who ruled our country for about 40 years

B. A trader from Afghanistan who came to study the dryfruit markets of our country.

C. A traveller from Uzbekistan who wrote· a book which is helpful to know the past of our country

D. A traveller who travelled from Kashmir to Kanyakumari to study the culture of Indian people.

 

Q. 64 The animals that flre awake at night can see .objects

A. in all colours

B. only in black and white colours

C. in green colour only

D. in red colour only

 

Q. 65 Select the correct statements about elephant herd :

A. An elephant herd has· mainly females and baby elephants upto 14-15 years old.

B. An elephant herd comprises members of a particular family.

C. The oldest female is the leader of the herd.

D. An elephant herd may accommodate any· number of female elephant and the young ones.

 

A. AandB

B. AandC

C. Band D

D. C and D

 

Q. 66 Inside the petals, in the middle of a flower we find a thin powdery structure, called 

A. Anther

B. Pollen

C. Radicle

D. Stigma

 

Q. 67 If you go to Ahmedabad (Gujarat) by train, then at Ahmedabad railway station you will find that most of the vendors are selling

A. Dhokla with chutney and lemon rice

B. Chholay-bhature and lassi

C. Idli-chutney and Vada-chutney

D. Puri-shaak and thanda doodh

 

Q. 68 The birds move their neck very often because

A. they can fly.

B. the birds eyes are fixed.

C. the birds have small eyes.

D. their ears are covered with feathers.

 

Q. 69 After diagnosis a doctor says to a patient that there is less haemoglobin in his blood. Which of the following should he eat to make up the deficiency of iron ?

A. Rice, sugar, amla

B. Green leafy vegetables, wheat, orange

C. Jaggery, lemon, pea

D. Amla, green leafy vegetables,• jaggery

 

Q. 70 National Curriculum Framework, 2005 strongly · recommends that teaching of EVS at primary stage should primarily aim at

A. developing understanding of basic concepts of the subject.

B. memorizing basic principles of the subject.

C. linking cIassroom learning to life outside the school.

D. acquiring skills to carry out· ·experiments independently.

 

Q. 71 Which one of the following is not . consistent with the requirements of EVS curriculum at primary stage ?

A. It should equip the learners with knowledge and skills to enter the world of work.

B. · It should inculcate in learners a concern for environment.

C. It should engage learners m acquiring methods and processes leading to generation of new knowledge.

D. It should suit cognitive level of the learners.

 

Q. 72 Higher priority and space has been given in NCERT textbooks on EVS to

A. explain basic concepts of the subject.

B. provide opportunities to learner for contemplation and wondering.

C. provide exact definitions of technical terms.

D. include large number of practice questions.

 

Q. 73 Which one of the following is not an objective of including poems and stories in EVS textbooks ?

A. To develop interest in the subject.

B. To have a change in routine and monotonous content

C. To provide fun and enjoyment for learners.

D. To promote imaginative and creative ability in the learners.

 

Q. 74 Good EVS curriculum should be ‘true to the child, true to life and true to the subject.’ Which of the following characteristics of a curriculum does not meet the above requirements ?

A. It promotes the value of freedom from fear and prejudice

B. It requires the Ieamer to view the subject as a social enterprise

C. It emphasises more on processes of teaching and learning

D. It emphasizes more on terms and definitions.

 

Q. 75 Nalini wants to introduce the topic on ‘Animals – our friends’ to Class III · students. In order to ·introduce the topic more interestingly, the best teaching strategy would be to 

A. use a chart showing pictures of different animals.

B. draw pictures of different animals on the blackboard.

C. show a video film on animals and their usefulness.

D. ask the students to see pictures of animals given in the textbook.

 

Q. 76 Class V NCERT EVS textbooks · include a section ‘what we have learnt’ in the end of every chapter. It is suggested that answer to questions included in this section should not be assessed in terms of. right or wrong. · · This shift has been done because

A. children cannot · write correct · · answers at this stage.

B. it enhances convenience to teachers in assessment.

C. it helps the teacher to know how children are learning

D. it reduces subjectivity in assessment.

 

Q. 77 Experimenting, Exploring, Investigating amd Questioning constitute essential elements· of activE learning of EVS. A teacher arranges the following activities related to the concept of ‘Food we should eat’.

Which one of the above four activities satisfy the condition for active learning?

A. Draws the diagrams of all such foods on the blackboard.

B. Gives examples of different foods each in essential components of food.

C. Asks students to collect relate information from. all possible sources.

D. Shows a video on the topic.

 

Q. 78 While teaching the topic on ‘Air is everywhere’ to Class IV students, Gitika. plans to perform the following activities :

Which one of the above proposed activities is not relevant for teaching of the topic

effectively ?

A. Take the students to a field trip.·

B. Explain the · concept . through specific examples.

C. Use multimedia capsule to explain the concept.

D. Ask probing questions related to the topic

 

Q. 79 The section on ‘Do this and find out’ included in different topics in EVS textbooks aim at 

A. providing direct expenences. hands-on

B. improving performance in examination

C. learning definitions of scientific terms

D. keeping the students engaged at home.

 

Q. 80 A good assignment in EVS. should primarily aim at

A. revise the lesson for effective learning.

B. ensure better utilization oftime.

C. keep the students engaged and disciplined.

D. provide extended opportunities.

 

Q. 81 As an EV.S teacher, the major objective of organizing a field trip to a zoo should be

A. to provide fun and enjoyment to students.

B. to have a change in monotony of routine teaching schedule.

C. to provide active learning experience to students

D. to satisfy parents on quality of education.

 

Q. 82 Which one of the following is not an objective of study of EVS in relation to Social Sciences ?

A. It should enable children to question the existing ideas and practices.

B. It should enable children to grow up as responsible member of society

C. It should enable children to respect differences of · culturalpractices.

D. It should enable children to learn correct definition of key terms

 

Q. 83 Kavita. wants to emphasise more on social issues like poverty, illiteracy and class inequalities in an EVS class. Which one of the following learning experiences will be more effective to achieve this objective ?

A. Organizing special lectures on related issues.

B. Asking the students to prepare charts on related social issues.

C. Asking the students to undertake group projects to collect and analyse related information.

D. Asking the students to write slogans on related issues.

 

Q. 84 Formative Assessment in EVS at primary stage does not include

A. identification of learning gaps of students.

B. identification of deficiencies in teaching.

C. enhancement of students’ learning.

D. grading and ranking of students.

 

Q. 85 Given below . are some human activities :.

A. Digging of mines.

B. Constructing dams.

C. Collection of leaves and herbs to sell them in the market.

D. Weaving baskets from bamboo.

E. Making leaf plates out of fallen leaves.

The activities responsible for the disappearance of forests is/are

 

A. only A

B. A and B both

C. A,BandC

D. B,C,DandE

 

Q. 86 Select correct statements are out Bronze:

A, Bronze is an element like iron, silver and gold.

B. Bronze is made on melting copper and tin.

C. Bronze is very strong and is used in ·making cannons and statues.

D. The utensils made of Bronze are lighter and stronger than that of aluminium.

 

A. B and C

B. C and D

C. D and A

D. A and C

 

Q. 87 A person living in Gandhidham · (Gujarat) wants to visit first Bhopal (Madhya Pradesh) and then Hyderabad (Andhra Pradesh). The directions of his journey will be

A. First towards East and then towards South

B. First towards West and then towards South

C. First towards South and then towards West

D. First towards South and then towards East

 

Q. 88 On a city map it was mentioned “Scale 1 em = 110 metres”. If the distance between two localities on the map is 15 em, then the actual distance between the two localities is 

A. 1165 centimetres

B. 1100 metres

C. 1500 metres

D. 1.65 kilometres

 

Q. 89 Mount Everest is a part of

A. India

B. Tibet

C. Nepal

D. Myanmar

 

Q. 90 Study the following duties/ responsibilities

A. Help others in carrying their bags.

B. Let the group follow you and · keep at the front.

C. Ask those to stay back who cannot climb properly.

D. Look after those who are ··not well and arrange food for the group.

E. Find a good place to stop and rest.

Which of the above are the responsibilities of a group leader in mountaineering ?

 

A. A, Band C

B. B, C and D

C. C, D and E

D. A,D and E

 

Questions: 91 – 99

Read the given passage and answer the questions that follow (Q: Nos. 91 to 99) by selecting the most appropriate option :

Why do poets use poems to tell aboutsocial injustices ? Tht: answer is simple.  This way a poet can catch and hold the reader’s attention, his/her emotions. Usually poets in their works present facts in order to capture attention of many people. These are not new facts that are presented to an audience. Poems are always aimed at reaching feelings of people and, thus, pulling strings. Literature of every state shows all the complexity of every epoch. When the situation is the same at several countries, it has a worldwide significance. Before talking about poetry, we should answer the question:

What is poetry ? Poetry is a special way of describing situations, things, ideas, feelings. ·Poets present their ideas in short phrases.

A poem can be .compared to a photograph as it reflects real .life, real situations and feelings, In a poem a poet captures the exact moment and represents it the way he/she has seen it. When you read a poem you see the poet’s subjective evaluation of facts, situations and the epoch in general. Poets of the Romantic Movement wrote their poems to share their feelings. They wrote to help people: 

understand their time from the poet’s point of view

 

Q. 91 A poem reflects the culture of the

A. period it belongs to

B. historical past

C. imaginary life

D. present only

 

Q. 92 Here, the expression ‘pulling strings’ means

A. challenging beliefs

B. promoting popular notions

C. secretly controlling thoughts

D. exerting strength

 

Q. 93 A word in the passage which means a quality of being intricate is

A. general

B. photograph

C. complexity

D. evaluation

 

Q. 94 Romantic . Movement is a literary period when the poetry mostly

A. is religious and philosophical

B. dealt with the supernatural and violence

C. focused on self-reliance and independence

D. emphasised on emotion and “imagination

 

Q. 95 The structure of poetry is usually characterized by

A. sequences of two or more words without an ‘action word’

B. long winded sentences

C. lengthy descriptive stanzas

D. short, factual stanzas

 

Q. 96 An example of ‘subjective evaluation’ is

A. Everybody likes dishes prepared with potatoes.

B. The potato is a staple in some countries.

C. There are over 25 types of potatoes around the world.

D. The potato has a large percentage of starch content.

 

Q. 97 The meaning of the word ‘epoch’ is a

A. lengthy, complex poem

B. type of literary work in England

C. distinctive point of time

D. being great and impressive

 

Q. 98 The purpose of poetry is to ___ _ the reader.

A. distract

B. appeal to

C. disturb

D. confuse

 

Q. 99 The themes of poems are usually about

A. everyday happenings

B. heroism and death

C. romance

D. important life events

 

Questions: 100 – 105

Read the given poem and answer the questions that follow (Q. Nos. 100 to 105) by selecting the most appropriate option.

Between the Miles

Because existence can become severe

in one d_ay,

just sense me and I’ll be there.

In the mind’s eye,

I’m not so far away.

If you hold o~t your hand,

in the-whispers,

I’ll become the zephyr …

and besiege you.

If your eye’s upon the stars,

in the crystalline darkness,

I’ll become the moon.

And the light shall guide you.

If you rest upon the ground,

in the warmth,

I’ll become the grass.

And embrace you.·:,.

If you tum outside,

in the wetness,

I’ll become the rain.

An upon your forehead, kiss you.

If you free the air,

in the light of day,

I’ll become the sun.

And smile for you.

Between the miles- ·

if you need me.

If you need a friend.

Let me be the friend, I want to be. ·

Heather Stoop

 

Q. 100 ‘The’ zephyr’ is a

A. fine quality of cloth

B. scent or odour

C. strong stream of air

D. gentle, mild breeze

 

Q. 101 An example of a metaphor is

A. ‘I’ll become the grass.’

B. ‘Between the miles’

C. ‘I want to be.’

D. ‘If you rest upon the ground’

 

Q. 102 A synonym of the word ‘besiege’ is

A. surround

B. attack

C. befriend

D. trap

 

Q. 103 The theme of the poem is about

A. separation

B. relationship

C. travel

D. nature

 

Q. 104 The ‘crystalline darkness’ that surrounding is

A. black and sombre

B. in the moonlight

C. lit up by the stars only

D. pitch dark and quiet

 

Q. 105 In the poem, the poet suggests that friendship is unaffected by

A. individual independence

B. changing feelings

C. time and distance

D. differences in attitude

 

Q. 106 A learner’s competence in English will improve when she/he receives or· learning experience that is appropriate.

A. an equal level

B. a slightly higher level

C. a range of levels

D. a slightly easier, lower level

 

Q. 107 A ‘mental block’ associated with English language learning is

A. interest in Sports and Arts

B. disinterest in studies in generaL

C. lack of opportunities to use English

D. dyslexia-a disability.

 

Q. 108 A company labels its frozen snacks 75% fat free rather than contains 25% fat so that people will view them more positively. This is an example of a

A. prototype

B. phoneme

C. subjective utility

D. semantic slanting

 

Q. 109 When children first start to speak in sentences, their speech may be described as

A. babbling

B. exceptionally soft

C. telegraphic

D. multi-lingual

 

Q. 110 Which observation supports Noam Chomsky’s ideas about language acquisition ?

A. Children’s language development follows a similar pattern across cultures.

B. The stages of language development occur at about the same ages in most children

C. Children acquire language quickly and effortlessly.

D. All of these

 

Q. 111 What is the system of rules that governs how words can be meaningfully arranged to form phrases and sentences ?

A. Language

B. Syntax

C. Morpheme·

D. Phoneme

 

Q. 112 A twelve-year-old child enjoys using puns. This enjoyment indicates that she has 

A. semantic slanting

B. deductive reasoning

C. mental blocks

D. metalinguistic awareness

 

Q. 113 Did you really ________ the money yesterday?

A. knead

B. kneed

C. need

D. needed

 

Q. 114 You don’t smoke, ________ ? (tag question)

A. have you

B. are you

C. do you

D. don’t you

 

Q. 115 A ‘critical period’ during language learning is

A. the period during which language can be acquired with greater ease than any other time.

B. the length of time before a comprehensive assessment . takes place in class.

C. best preparatory period for any language project.

D. special time set aside for students to intensively practice language use.

 

Q. 116 The two skills required to take notes effectively are

A. using symbols and abbreviations instead of words.

B. re-writing a text, using your own wOrds

C. writing legibly with correct punctuation.

D. writing fluently, using conjunctions.

 

Q. 117 Remediation, when students find difficulty in the use of different ‘modals’ would be for them to

A. be given ample practice in using modals in a set of sentences.

B. frame sentences on their own and teacher corrects them.

C. learn about the structures outside the classroom through suitable activities.

D. practice by collaboratively completing tasks where structures are used integratively, in a variety of real life situations.

 

Q. 118 What is wrong with the following multiple choice question ?

Tick the most appropriate : The Metro theatre is located __ _ LodhiRoad.

(a) over

(b) at

(c) beside

(d) behind

 

A. All answers are wrong

B. Two are wrong

C. The statement IS not correctly framed

D. ‘Over’ is the correct answer

 

Q. 119 Receptive Language skills are

A. using grammatical structures accurately.

B. writing in a range of styles.

C. listening and reading for information.

D. being able to self-correct while using language.

 

Q. 120 The ‘Natural Order’ in the process of learning English suggests that, children

A. are slow at learning to speak when riot in school.

B. learn to read and write simultaneously.

C. are able to speak first and then listen

D. acquire some language structures earlier than others.

 

Questions: 121 – 129

Read the given passage and answer the questions that follow (Q. Nos. 121 to 129) by selecting the most appropriate option

1. The fossil remains of the first flying vertebrates, the pterosaurs, have intrigued paleontologists for more than two centuries. How such large creatures, which had wingspans from 8 to 12 metres, solved the problems of powered flight, and exactly what these creatures were-reptiles or birds – are among the questions scientists have puzzled over

2.Perhaps the least controversial assertion about the pterosaurs is that they were reptiles. Their skulls, pelvises and hind feet are reptilian. The anatomy of their wings suggests that they did not evolve. into the class of birds. In pterosaurs, a greatly elongated fourth finger of each forelimb supported a wmg like membrane. In birds the second finger is the principle strut of the wing. If the pterosaur walked or remained stationary, the fourth finger and with it the wing, could. only tum upward in an extended inverted V -shape alongside of the animal’s body. Both the pterosaurs and the birds have hollow bones, a feature that represents a saving in weight. In the birds, however, these bones are reinforced more massively by internal struts.

3.Although scales typically cover reptiles, the pterosaurs probably had hairy coats. The recent discovery of a pterosaur specimen covered in long, dense and relatively thick hair-like fossil material, was the first clear evidence that this reasoning was correct. Efforts to explain how the pterosaurs became air-borne have led to suggestions that they launched themselves by jumping . from cliffs, by dropping from trees, or even by rising into light winds from the crests of waves.

 

Q. 121 The skeleton of a pterosaur can be distinguished from a birds by the

A. size if its wing span

B. presence of hollow bones

C. hook-like projections at the hind feet.

D. the anatomy of its wing span.

 

Q. 122 Which is the characteristic of pterosaur?

A. They hung upside down like bats before flight

B. Flew to capture prey

C. Unable to fold their wing fully at rest

D. Lived mostly in the forest

 

Q. 123 The elongated finger in the ___ _ supported the outstretched wings.

A. pterosaurs

B. birds

C. both

D. neither

 

Q. 124 The · body of the pterosaurs was covered in

A. feathers

B. scales

C. fur

D. smooth skin

 

Q. 125 The pterosaurs flew by

A. jumping off a mountain ledge

B. pushed by wind before take of

C. jumping upwards with force

D. moment.um gained by running

 

Q. 126 A synonym for ‘compressed’, from the passage is

A. launch

B. dense

C. light

D. strut

 

Q. 127 The opposite of ‘contrqversial’ Is

A. questionable

B. uncertain

C. debatable

D. undisputed

 

Q. 128 It can be understood from the passage that scientists believe that the

A. large wings help pterosaurs to. fly great distances.

B. hollow bones showed they evolved from bats.

C. fossil remains explain how they flew

D. pterosaurs walked on all fours.

 

Q. 129 Fossils often left scientists in doubt whether the pterosaur

A. ever existed at all

B. how many lived at that period

C. their size and weight

D. their shape and gender

 

Questions: 130 – 135

Read the given passage and answer the . questions that follow (Q. Nos. 130 to 135) by selecting the most appropriate option :

1.A man found a cocoon of a butterfly. One day a small opening appeared. He sat and watched the butterfly for several hours as it struggled to force its body through that little hole. Then it seemed to stop making any progress. It appeared as if it had gotten as far as it could, and it could go no further: So the man decided to help the butterfly. He took al pair of scissors and snipped off the remaining bit of the cocoon. ‘fh.e butterfly then emerged easily. But it had a swollen body and small, shriveled wings. The man continued to watch the ‘butterfly because he expected that, . at any moment, the wings would enlarge and expand to be able to support the body, which would contract in time

2.Neither happened! In fact, the butterfly spent the rest of its life crawling around with a swollen body and shriveled wings. It never was able to fly. What the man, in his kindness and haste, did not understand was that the restricting cocoon and the struggle required for the butterfly to get through the tiny opening were God’s way of forcing fluid from the body of the butterfly into its wings so that it would be ready for flight once it achieved its freedom from the cocoon. 

 

Q. 130 The man’s first instinct was

A. leave the cocoon alone

B. help the butterfly

C. leave the butterfly alone

D. keep watching

 

Q. 131 The natural process would have the wings of the ·butterfly

A. unfold and stretch out

B. fold up and remain snug

C. half open and snug against the body

D. unfold.and remain stiff

 

Q. 132 A word that means ‘to make or become withered’ is

A. moistened

B. folded

C. wasted

D. shrivelled

 

Q. 133 The writer’s message in his/her essay is about

A. not to have any problems

B. need for struggles in life

C. escape pain at any cost

D. needless struggles in life

 

Q. 134 The essay is ____ in form. 

A. factual

B. descriptive

C. discursive

D. argumentative

 

Q. 135 A man noticed that the—‘——

A. butterfly was hidden

B. cocoon was growing

C. cocoon was moving

D. butterfly was e’merging

 

Q. 136 The ‘bottom up model’ of curriculum is one where

A. learning IS based on a set of software to make curriculum more Reamer friendly.

B. the curriculum that allows freedom for student mobility with increased choice of curricular activity and encourages learning by doing.

C. the learning process 1s geared towards career orientation.

D. a need-based distance education with indirect influence on students.

 

Q. 137 The Humanistic Approach is specifically tuned to the

A. mastery of academic disciplines with all their characteristic features .

B. application of learnt structure, content, concepts and principals to new situations.

C. processes that enable students to discover structures for themselves

D. process where sequence is taught along with how to present the related .contents .

 

Q. 138 A ‘special needs language classroom’ is ideally

A. exclusively furnished.

B. located separately.

C. integrates all types of learners.

D. has extra teachers to help regular teachers.

 

Q. 139 One of the challenges of ‘Behaviour Management’ in a senior class is

A. students’ readiness to use the smart board.

B. student’s lack of self study skills.

C. teachers’ preference to conduct group rather than individual work

D. teachers’ lack of self confidence.

 

Q. 140 To inculcate a ‘Never Give Up . Attitude’, a suitable activity is the one when students 

A. sang two popular songs and exhibited some of their art and craft works during the parent teacher meet.

B. ·made modifications to their paper planes and tested them. again, experimented with the best way to get them to go the distance and shared their finding.

C. in groups created graphs about the difficult situations that students have had to face in life.

D. managed to get the Principal’s permission to go out and play during the English period.

 

Q. 141 Assessing reading at Class VII, can be done most effectively through a

A. spoken qUiz based on the meanings of words and expressions.

B. written test based the characters and events in the story/text.

C. an oral interview to find out how much they have read.

D. writing a 50 word book/text _review as a small project.

 

Q. 142 Curriculum development follows the following sequence :

A. Formulation of objectives, assessment of needs, selection of texts/learning experiences, evaluation

B. Selection of texts/learning experiences, assessment of needs, formulation of objectives, evaluation

C. Assessment of needs, formulatjon (4) of objectives, selection of texts/learning expenences, evaluation of objectives, selection of texts/learning expenences, evaluation

D. Formulation of objectives, assessment of needs, evaluation, selection of texts/learning experiences

 

Q. 143 The learning experiences that offer a vicarious experience to learners are

A. real objects and specimens

B. abstract words, case study

C. display boards, film clips

D. field trips, observations

 

Q. 144 In Computer Aided Instruction [CAl), the ‘simulation mode’ is where learners

A. experience real life systems and phenomena.

B. receive bits of information followed by questions with immediate feedback.

C. a senes of exercises with ‘ repetition practice

D. get problems which are solved by a process of trial and error.

 

Q. 145 A ‘listening stimulus’

A. presents input to separate groups of students who gather again to share what they listened

B. presents an information ‘ gap activity such as giving directions.

C. is listening to a good commentary to review it.

D. enables students to discuss a set of criteria which they prioritize to complete and present a task.

 

Q. 146 The ‘interactional routine’ during speaking assessment includes a

A. negotiating meanings, taking turns and allowing others to take turns.

B. describing one’s school or its environs informally

C. ‘telephone’ conversation with another.

D. comparing two or more objects/places/event for the assessor

 

Q. 147 Retrieval skills in writing are

A. note making and note taking

B. diagramming and summarising

C. abilities to do extensive reference work

D. organizing information ·while reading/listening

 

Q. 148 ‘Awareness raising’ grammar· games encourage students to

A. think consciously about the structures they have learnt.

B. collaborate in completing a given activity.

C. engage and feel about human relationships while the teacher controls the structures.

D. use learnt structures to communicate with one another about a given theme.

 

Q. 149 Language acquisition

A. is the memorization and use of necessary vocabulary

B. involves a systematic approach to the analysis and comprehension of grammar as well as to the memorization of vocabulary.

C. refers to the process of learning a native or a second language because of the innate capacity of the human brain.

D. is a technique intended to simulate the environment in which children learn their native language.

 

Q. 150 Noah Chomsky’s reference to “deep structures” means a

A. hidden set of grammatical rules learnt through intensive study.

B. transformational grammar that -has led in tum to increased interest in comparative linguistics.

C. a trend that English is the most common-auxiliary language in the world.

D. universal wammar underlying all languages and corresponding to an innate capacity of the human brain

 

Answer Sheet 
Question123456 78910
AnswerBCDAAABCAC
Question11121314151617181920
AnswerDAACDABDCB
Question21222324252627282930
AnswerDDABCCCBBA
Question31323334353637383940
AnswerCCABBABDBC
Question41424344454647484950
AnswerDDABCCCABD
Question51525354555657585960
AnswerADAACACCBA
Question61626364656667686970
AnswerDDCBBBABDC
Question71727374757677787980
AnswerABCDCCCAAA
Question81828384858687888990
AnswerCDCDBAADBD
Question919293949596979899100
AnswerAACDAACBDD
Question 101102103104105106107108109110
AnswerAABDCCCDAD
Question 111112113114115116117118119120
AnswerBDCCAADDCB
Question121122123124125126127128129130
AnswerDCACADDACB
Question131132133134135136137138139140
AnswerADBCDBCCBB
Question141142143144145146147148149150
AnswerCCCACCBCBD
×

Hello!

Click one of our representatives below to chat on WhatsApp or send us an email to info@vidhyarthidarpan.com

×